[Ответить в тред] Ответить в тред



<<
Назад | Вниз | Обновить тред | Автообновление
522 | 78 | 174

Тред тупых вопросов №83 Аноним 24/01/18 Срд 21:24:04  373250  
DLZ7X4rWkAc7c25.jpg (85Кб, 706x765)
PlanckCardFront.gif (563Кб, 1024x742)
videoplayback ([...].mp4 (32058Кб, 640x360, 00:12:13)
wallpapersden.c[...].jpg (165Кб, 1600x900)
Тред вопросов о жизни, Вселенной и всем таком.

Спрашиваем то, за что в других местах выдают путёвку в биореактор. Здесь анонимные ученые мирового уровня критически рассмотрят любые гениальные идеи и нарисованные в Paint схемы.

Предыдущий тут: https://2ch.hk/spc/res/367033.html

Q: Можно быстрее?
A: Можно упасть в пузырь альбукерке, наса уже почти надула его.

Q: Я начитался охуительных историй про уфологию, че делать, нам жопа?
A: Да, тебе жопа, можешь сгонять в зогач или куда оттуда пошлют.

Q: Что будет с человеком в вакууме без скафандра / если он упадет на черную дыру / попробует ступить на поверхность газового гиганта/солнца?
A: Он умрёт.
Аноним 24/01/18 Срд 21:57:51  373285
>>373250 (OP)
Допустим, правнуки сумели запустить на орбиту реактор с мощностью на уровне ВВЭР-1200. Смогут они долететь до соседней звезды и вернуться? А до Плутона?
Аноним 24/01/18 Срд 22:14:33  373294
hawking.jpg (88Кб, 932x797)
>>373250 (OP)
>упасть в пузырь альбукерке
Ебанный стыд...
Во-первых, Алькубьерре.
Во-вторых, не упасть, а создавать вокруг корабля изнутри (иначе кина не будет).
В-третьих, НАСА искривляет пространство на десятимиллионную часть, контролируя это сверхточными интерферометрами, до самого варп-привода здесь - как до Антарктиды раком.
Аноним 24/01/18 Срд 22:43:19  373308
Почему произошел большой взрыв?
Вся хуйня была сжата в очень плотный шар.
По сути, это была сингулярность черной дыры.
А, как мы знаем, черные дыры не взрываются почти.
А тут просто сверх-еба плотная хуйня и взорвалась
Аноним 24/01/18 Срд 22:50:13  373311
Возможен ли запил твердотопливной ракеты которая сможет в орбиту?

Если боковой бустер от шатла запустить без груза, далеко улетит?
Аноним 24/01/18 Срд 22:52:05  373312
>>373311
МБР так-то часто твердотопливные
Аноним 24/01/18 Срд 23:03:09  373318
europe-successf[...].jpg (51Кб, 1024x576)
>>373311
>Возможен ли запил твердотопливной ракеты которая сможет в орбиту?
Minotaur и Vega недоумённо всматриваются в твой вопрос.
>Если боковой бустер от шатла запустить без груза, далеко улетит?
Очень далеко, из него пытались сделать ракету, да и сейчас пытаются, но там уже не совсем шаттловская конструкция.
>>373312
МБР же на орбиту не выходят, им-то зачем.
Аноним 24/01/18 Срд 23:08:28  373322
>>373318
> МБР же на орбиту не выходят, им-то зачем.
Ну я к тому, что летают же заебись, управляются хорошо, высоко и на большой скорости.
Если надо будет на орбиту запульнуть - плевое дело.

Ну ты сам уже ответил на вопрос
Аноним 24/01/18 Срд 23:32:47  373332
Тащемта МБР еще как летают
>w: Например, при помощи американских МБР Атлас и Титан осуществлялись запуски космических кораблей Меркурий и Джемини. А советские МБР PC-20, PC-18 и морская Р-29РМ послужили основой для создания ракет-носителей Днепр, Стрела, Рокот и Штиль.
Аноним 24/01/18 Срд 23:38:31  373337
>>373332
Это и так все знают, а ты ещё и Р-7 забыл из википедии скопировать.
Имеется ввиду что ракета с ядерным хуйцом не выходит на орбиту, а не что она не может этого сделать. Тем более что гражданские версии обычно хотя бы немного отличаются, а иногда и довольно сильно.
Аноним 24/01/18 Срд 23:48:55  373343
>>373318
>Очень далеко, из него пытались сделать ракету, да и сейчас пытаются, но там уже не совсем шаттловская конструкция.

Подробнее, и можно ли сделать одноступенчатую в теории?
Аноним 25/01/18 Чтв 00:01:52  373345
NGL-Scale3.jpg (801Кб, 2632x2055)
>>373343
>Подробнее
Вот что было:
https://www.wikiwand.com/en/Ares_I
https://www.wikiwand.com/en/Liberty_(rocket)
Вот что сейчас пилят:
https://www.wikiwand.com/en/Next_Generation_Launcher
>можно ли сделать одноступенчатую в теории
Можно конечно, даже были попытки:
https://www.wikiwand.com/en/Single-stage-to-orbit
Аноним 25/01/18 Чтв 00:23:32  373353
Возможно ли использовать мобильные пусковые комплексы ядерных пинусов, типа Тополя или Молодца для запуска на низкие, но при этом круговые орбиты хоть какого-нибудь микроспутникового хлама? Не важно, что он сгорит спустя раз-два-три витка. Если сумеет сделать какие-нибудь снимки и передать их на Землю, то это уже можно считать успехом.
25/01/18 Чтв 00:32:04  373356
3e28d6f2a506cbd[...].jpg (1228Кб, 1940x2912)
b40c419492e1168[...].jpg (728Кб, 1700x1235)
8094a5b7a803a85[...].JPG (919Кб, 2265x3304)
>>373353
Уже запускали.
25/01/18 Чтв 00:32:32  373357
3bc86bed6369c1d[...].jpg (951Кб, 2768x1768)
57fd5eea5bcaee6[...].jpg (1547Кб, 3310x2247)
ee42ab8a7ba5343[...].jpg (373Кб, 1597x1789)
>>373353
Аноним 25/01/18 Чтв 01:05:39  373367
>>373357
>>373356
Это ладно, за Россией еще одно по своему уникальное достижение держится: запуск на орбиту спутников с борта подводной лодки. И в надводном положении, и в подводном.
Аноним 25/01/18 Чтв 01:09:16  373368
>>373367
Эээ... Ну я ещё могу понять, зачем можно запустить спутник с подводной лодки, но из подводного положения-то нахуя?
Аноним 25/01/18 Чтв 01:09:22  373369
>>373337
https://ru.wikipedia.org/wiki/Р-36орб
Аноним 25/01/18 Чтв 01:32:33  373375
>>373308
Бамп!
Аноним 25/01/18 Чтв 05:35:15  373385
>>373308
>А тут просто сверх-еба плотная хуйня и взорвалась
>А тут просто
У дурачков вроде тебя, всегда все просто
Где-то что-то куском прочел, либо слышал краем уха, и понимает буквально. Взрыв, ага, значит что-то в каком-то объеме взорвалось. На большее интеллекта хлебушка просто не хватает. Начать хотя-бы с того что, большой взрыв, в первую очередь, это рождение пустоты (нашего трехмерного пространства и времени) в нигде и из нихуя. И этот взрыв, появление трехмерной пустоты из нихуя, продолжается до сих пор.
Аноним 25/01/18 Чтв 07:48:48  373389
>>373285
До плутона запросто.
Вывод такой йобы подразумевает возможность вывода кучи тоннажа, а значит йоба-корабль можно запилить.
До соседней звезды уже нет - умрут по пути, надо корабль поколений и колоссальных размеров.
Аноним 25/01/18 Чтв 07:50:41  373391
>>373308
>Почему произошел большой взрыв?
Никто не знает, и возможно, что это узнать невозможно принципиально. Мы в этой вселенной с ее законами, что было до нее/за ее пределами с другими законами (?) можно только гадать.
А так - почитай Митио Каку или просто хотя бы глянь PBS SpaceTime
Аноним 25/01/18 Чтв 07:54:24  373392
>>373311
>Возможен ли запил твердотопливной ракеты которая сможет в орбиту?
Джапы свой спутник на четырхеступенчатом твердотопливном Ниссане и вывели. https://ru.wikipedia.org/wiki/Ламбда-4S

>>373318
>Очень далеко, из него пытались сделать ракету, да и сейчас пытаются, но там уже не совсем шаттловская конструкция.
https://ru.wikipedia.org/wiki/Арес-1
>МБР же на орбиту не выходят, им-то зачем.
https://ru.wikipedia.org/wiki/Р-36орб
>>373322
>Если надо будет на орбиту запульнуть - плевое дело.
Ты только что конверсионные ракеты.
Аноним 25/01/18 Чтв 08:16:10  373397
Почему при увеличении УИ падает тяга? Вот допустим ттрд - самая большая тяга, но низкий УИ. У вакуумных жрд тяга ниже чем у маршевых для первой ступени, но УИ выше. У ионников тоже и так до фотонных, где УИ максимально возможный, но тяга вообще мизерная
Аноним 25/01/18 Чтв 08:24:11  373402
>>373397
Потому что E=(mv^2)/2, а mv=MV. Энергии на борту одинаково, так что при увеличении скорости истечения приходится уменьшать массу выхлопа квадратично, что ведет к уменьшению импульса и соответственно тяги.
Аноним 25/01/18 Чтв 08:56:58  373405
>>373294
Спасибо, что освятил.
Аноним 25/01/18 Чтв 08:58:37  373406
Можно ли узнавать состав астероидов не прилетая к ним и не буря их?

>>373405
Двачую. Теплеет на душе, когда традиции соблюдены.
Аноним 25/01/18 Чтв 09:44:00  373415
>>373368
Чтоб все охуели, как мы можем.
Аноним 25/01/18 Чтв 09:46:09  373416
>>373415
Топ кек. Я точно так же хотел ответить за неимением лучшего объяснения, но не стал.
Аноним 25/01/18 Чтв 10:21:41  373426
>>373385
>нихуя
Восставим же великое нихуя!
Аноним 25/01/18 Чтв 10:22:40  373428
>>373426
>Восставим
Вставим Восславим Фикс
Аноним 25/01/18 Чтв 11:07:08  373453
Можно ли было сделать термозащиту шаттлов не из плиточек, а из более крупных кусков?
Аноним 25/01/18 Чтв 11:15:38  373454
>>373453
Большие куски сложнее производить, сложнее заменять, дороже опять же.
Аноним 25/01/18 Чтв 11:55:28  373461
Вояджеры разгоняли о Юпитер. Правильно я понимаю что они находятся на его мегавытянутой орбите?
Аноним 25/01/18 Чтв 11:58:44  373465
>>373461
Нет, неправильно.
Когда орбита незамкнутая то ты на пролетной траектории. Орбита это если бы они вернулись к нему, но они никогда не вернутся ни к нему ни к солну
Аноним 25/01/18 Чтв 12:03:18  373469
>>373465
Получается они не на чьей орбите? А когда они попадут под действие гравитации звезды когда прилетят в Альфу-Центавру (или куда они там летят) останутся на её орбите?
Аноним 25/01/18 Чтв 12:05:28  373470
>>373469
На орбите вокруг галактического центра.
Читай про 1 2 3 4 космические скорости.
https://ru.wikipedia.org/wiki/Первая_космическая_скорость
https://ru.wikipedia.org/wiki/Вторая_космическая_скорость
https://ru.wikipedia.org/wiki/Третья_космическая_скорость
https://ru.wikipedia.org/wiki/Четвёртая_космическая_скорость
https://ru.wikipedia.org/wiki/Задняя_космическая_скорость
Аноним 25/01/18 Чтв 12:09:11  373473
>>373406
> Можно ли узнавать состав астероидов не прилетая к ним и не буря их?
Спектрография в отражённом свете даёт какие-то результаты. Ещё альтернатива бурению - уронить на него что-нибудь тяжёлое и посмотреть спектр фонтанчика пыли.
Аноним 25/01/18 Чтв 12:09:33  373474
>>373470
>Задняя_космическая_скорость
сук)))

Благодарю за отзыв!
Аноним 25/01/18 Чтв 12:11:29  373475
>>373473
А что будут и как делать? Вот я, допустим, миллиардер и хочу стать еще миллиардеристей. Основал свою компанию SpaceY, успешно могу в ракеты уже. Хочу стать осмиевым магнатом, где и как мне именно осмий искать?
Аноним 25/01/18 Чтв 12:26:50  373482
tumblrm66rirVuQ[...].jpg (187Кб, 800x533)
Какая нужна скорость, чтобы покинуть чёрную дыру? Берём расчёт от неглубого нырка под поверхность горизонта событий до отрыва от материальной точки в центре масс дыры.

И да, буббльгум Альбукерке позволит нырять в чёрную дыру? А проходить сквозь звёзды, юпитеры, нейтронки, земли и астероиды без последствий она поможет? Или литосферное торможение неминуемо даже для пузыря Альбукоина?

А что былобы видно при полёте в пузыре из лобового стекла шаттла? Что-нибудь было бы видно?
Аноним 25/01/18 Чтв 12:31:42  373484
>>373482
>Какая нужна скорость, чтобы покинуть чёрную дыру?
Недостижимая, выше скорости света.
>Берём расчёт от неглубого нырка под поверхность горизонта событий до отрыва от материальной точки в центре масс дыры.
Неважно, попав под горизонт уже не выбраться.
>И да, буббльгум Альбукерке позволит нырять в чёрную дыру?
Нет, он возможно сможет ее горизонт немного оттаскивать/подтаскивать.
>А проходить сквозь звёзды, юпитеры, нейтронки, земли и астероиды без последствий она поможет?
Нет, материя от пузыря никуда не девается, пузырь в том же пространстве, просто оно искажено спереди и сздаи.
>А что былобы видно при полёте в пузыре из лобового стекла шаттла? Что-нибудь было бы видно?
Да.
Я где-то видел видео с искажением из-за пузыря от первого лица, но найти не могу.
Аноним 25/01/18 Чтв 12:41:09  373490
scr00015.jpg (262Кб, 1920x1080)
>>373484
> Я где-то видел видео с искажением из-за пузыря от первого лица
В space engine кстати такое есть, но не уверен насчёт реалистичности
Аноним 25/01/18 Чтв 12:42:14  373491
>>373490
О какой реалистичности может идти речь когда мы говорим о выдуманном концепте который только на бумаге?
А так - вполне верно, ведь там расчеты исходя из именно искажений самого пузыря.
Аноним 25/01/18 Чтв 12:49:11  373492
>>373490
Вот видео https://www.youtube.com/watch?v=Ph4GlrsijDk
Аноним 25/01/18 Чтв 12:50:56  373494
>>373492
Обкончался. Теперь это фап-тред?
Аноним 25/01/18 Чтв 14:32:52  373511
>>373475
Ну в спектре отражённого света можно примерно почувствовать. Потом пускаешь зонд, который хуячит астероиды и смотришь спектры в пыли.
Аноним 25/01/18 Чтв 14:36:09  373512
>>373511
Слабенько как-то.
Я думал, какие-нибудь рентгены и лазоры будут.
Аноним 25/01/18 Чтв 14:47:21  373513
>>373512
Ну можешь изобрести лазер в гамма-диапазоне и просвечивать им астероиды. Только нужно как-то так расположить излучатель и приёмник, чтобы можно было много астероидов просвечивать. Например, зонд с лазером может лететь по орбите за поясом астеоридов в противоположную сторону, но всё равно тогда он сможет просвечивать только те астероиды, которые ровно в плоскости его орбиты. Ну или лучше лазер около земли, на луне например, а летит приёмник.
Аноним 25/01/18 Чтв 15:07:08  373515
>>373470
>https://ru.wikipedia.org/wiki/Задняя_космическая_скорость
А где космическая нейтралка?
Аноним 25/01/18 Чтв 15:12:31  373518
>>373515
Держи.
https://ru.wikipedia.org/wiki/Нейтральная_космическая_скорость
Теперь ты можешь собрать механическую коробку космических передач.
Аноним 25/01/18 Чтв 15:47:00  373529
1398236830008.jpg (43Кб, 332x500)
Тупой вопрос от дебила, возможно платина.

Какие есть ресурсы на Луне и Марсе дальше заглядывать бессмысленно? потенциально подходящие для добычи там и использовании в какой либо промышленности на Земле? Там есть что то такое, что бы бы весьма востребовано тут, но на нашей уютной планете этого очень мало и нужно больше?
Аноним 25/01/18 Чтв 16:01:21  373534
>>373529
На Луне дохуя всего, на астероидах тоже есть.
Аноним 25/01/18 Чтв 16:13:07  373537
>>373529
Редкоземельные металлы, как правило, из того что может себя окупить. На железных астероидах их точно много. На Луне в основном все маняфантазируют про Гелий-3.
Аноним 25/01/18 Чтв 16:28:09  373542
>>373537
Мун Экспресс хотят добывать ниобий, иттрий и диспрозий.
Аноним 25/01/18 Чтв 16:34:39  373545
14237678724073.jpg (106Кб, 510x510)
>>373529
Луна бохата литием-3 и серебром обыкновенным
Аноним 25/01/18 Чтв 16:48:48  373547
bCwKS-igh8.jpg (63Кб, 640x593)
АНОНЫ, ПОМОГИТЕ!
Ни на ютубе, ни в ВК нету аудиокниги Циолковского "Промышленное освоение космоса".
Может, вы знаете, где её искать?
Аноним 25/01/18 Чтв 16:55:47  373548
>>373545
>литием-3
Может быть гелием-3? Литий-3 не существует.
Аноним 25/01/18 Чтв 16:58:37  373550
>>373548
Это только на земле, на луне его полно!
Конечно, Гелий-3, которому нет применения.
Кстате, что за пиздёж про то, что гелий кончается? Четыре миллиарда лет не кончался и тут, блядь, паника, гроб-гроб-кладбище-пидор кончается, лол.
Аноним 25/01/18 Чтв 17:19:41  373553
>>373550
Его тупо мало. Он постоянно выделяется из недр земли и постоянно съебывает в космос. Добывать его в виде ископаемого экономически нецелесообразно. Гораздо проще его производить искусственно, в реакторах и ускорителях частиц. На Луне Гелий-3 сконцентрирован в тонком слое реголита, то есть выкопать там карьер и добывать его в одном месте не получится. Учитывая, что для получения 1 тонны гелия-3 нужно перелопатить 100-150 млн тонн реголита, есть вообще большие сомнения, что цена добытого гелия-3 на Луне возможно будет сделать ниже, чем произведенного на Земле.
Аноним 25/01/18 Чтв 17:20:51  373554
>>373553
А как г3 может давать энергию, если для его получения требуется энергия? Звучит, как херня.
А водород-водородный термояд вообще возможен человеками?
Аноним 25/01/18 Чтв 17:27:59  373555
>>373554
>А водород-водородный термояд вообще возможен человеками?
Уже более полувека как освоен в токамаках и стеллаторах. Поддерживать пока не научились. Вот ИТЭР строят.

А вот использовать гелий-3 для этого даже толком не пробовали, нужна гораздо более высокая температура, чем привычная реакция T+D.
Аноним 25/01/18 Чтв 17:29:07  373556
>>373555
ИТЭР разве водородный? Я думал, там литий.
Еще слышал что-то про борную реакцию, но на фоне разговоров про ХТЯС, так что не внял всерьёз.
Аноним 25/01/18 Чтв 17:56:16  373557
>>373555
> освоен
Только если считать освоением Q (отношение вкачанной в плазму энергии к произведенной термоядом энергией) в районе единицы. Так-то даже ИТЭР (Q в районе 8-10) это не освоение, освоение будет в DEMO лет через 50. Или в водородных бомбах больше чем полвека как.
>>373556
> ИТЭР разве водородный?
Дейтерий + тритий. Литий у тебя всплыл, скорее всего, из-за планов на эксперименты по получнию из лития-6 гелия-3 в облучаемом нейтронами бланкете, такое на ИТЭР планируют провернуть, но это не основная реакция.
> Еще слышал что-то про борную реакцию, но на фоне разговоров про ХТЯС, так что не внял всерьёз.
Протон+Бор-11 это, на самом деле, "нормальный" горячий термояд, даже пиздец какой горячий и хрендостижимый в сравнении с D+T. Но имеет джва охуенных преимущества:
1. Бора на Земле хоть залейся, никаких проблем с топливом, это вам не Гелий-3
2. Околонулевой нейтронный выхлоп, соответственно конструкции реактора выходят без наведённой радиации, что сильно упрощает материаловедение и пиздец как упрощает отношения с атомнадзорами.
Потому физики с инженерами на него потихоньку пофапывают, но всерьёз замахивается на такие реакторы только один пиндостанский стартап.
Аноним 25/01/18 Чтв 18:00:03  373559
>>373557
Спасибо, очень познавательно. Если моар понапишешь интересных фактов, будет круто.
Аноним 25/01/18 Чтв 22:58:17  373631
>>373356
>>373357
Так какого же тогда хуя мы тратимся на "Восточный", если можем вот хоть прям из Подмосковья, хоть из самой Москвы запускать спутники?
Аноним 25/01/18 Чтв 23:00:46  373633
>>373631
Отработавшую ступень у себя в прихожей обнаружить не боишься?
Аноним 25/01/18 Чтв 23:11:03  373635
>>373633
А парашюты Керин зачем придумал?
Аноним 25/01/18 Чтв 23:22:22  373636
>>373631
Потому, что спутники что приносят деньги весят больше полутонны.
Аноним 25/01/18 Чтв 23:23:47  373637
DebrisStage1019[...].jpg (130Кб, 961x591)
>>373635
Какой ещё Керин?
Ну не знаю, наверное чтобы тащить лишний вес, снижая вес ПН, ради того чтобы неконтролируемо спускающаяся ебала приземлилась в рандомное место типа автодороги, огорода бабки сраки или беседку с мирно отдыхающим пьяным быдлецом. Зато запуск из подмосковья, которое менее выгодно расположено, чем Байка и Восточный.
И да, на такой ракете большую йобу не выведешь, поэтому и тратятся на Восточный.
Аноним 25/01/18 Чтв 23:25:10  373638
>>373637
это скайлон?
Аноним 25/01/18 Чтв 23:27:21  373639
>>373638
Нет, это Н-1.
Аноним 25/01/18 Чтв 23:29:43  373640
>>373637
Кербин. Хуй с ним.
Аноним 26/01/18 Птн 09:37:09  373681
>>373547
Идиот, первая ссылка в гугле.
Аноним 26/01/18 Птн 09:38:55  373682
>>373631
>Так какого же тогда хуя мы тратимся на "Восточный"
Можно несколько миллардов денег распилить.
Аноним 26/01/18 Птн 10:24:02  373688
Водород вообще стоит ебли с ним? Т.е. чуть больший импульс не херится огромными размерами баков и прочим, ну лишний вес и сопротивление воздуха? Какие там цифры выходят по факту, а не на бумаге?
Кстати почему американцы выбрали водород, а мы нет?
Аноним 26/01/18 Птн 10:28:18  373689
>>373688
Нам НЕУДОБНА было.
Потом же Энергию запилили.
У водорода тяга говеная при отличном импульсе потому все водородные ракеты с бустерами ТТРД (шуттел, СЛС, Ариан) или керосиновыми (Энергия).
Аноним 26/01/18 Птн 11:35:12  373700
image.png (2806Кб, 1179x1500)
>>373689
> потому все водородные ракеты с бустерами ТТРД (шуттел, СЛС, Ариан) или керосиновыми (Энергия).
Дельта-4 смотрит на тебя как на неосведомлённого человека.
Аноним 26/01/18 Птн 11:41:32  373703
Стикер (0Кб, 512x512)
>>373700
Как, блядь, я про неё забыл?
Аноним 26/01/18 Птн 12:22:34  373713
>>373688
>чуть больший импульс не херится огромными размерами баков и прочим
УИ у водорода не "чуть больший", а сильно больший, и размерами баков не херится. А вот прочими эффектами — вполне.

Во-первых, глубокая криогенность: водород имеет температуру кипения 20К (у кислорода 90K, у метана 111K). При температуре кипения водорода воздух превращается в твёрдое вещество. Термоизоляционная пена на внешнем баке шаттла -- именно из-за этого.

Во-вторых, у водорода крохотная молекула, которая норовит утечь через самую мелкую щель.

В-третьих, эффекты водородного охрупчивания https://en.wikipedia.org/wiki/Hydrogen_embrittlement
Аноним 26/01/18 Птн 12:24:23  373714
>>373713
>охрупчивания
Проиграл. Впервые услышал это слово, спасибо.
Аноним 26/01/18 Птн 14:47:00  373724
>>373250 (OP)
Почему российская космонавтика в жопе?
Аноним 26/01/18 Птн 14:54:25  373725
>>373724
Потому что такие долбоёбы как ты промахиваются мимо треда Роскосого. Нахуй иди.
Аноним 26/01/18 Птн 15:55:33  373733
>>373713
>>373688

>Во-вторых, у водорода крохотная молекула, которая норовит утечь через самую мелкую щель.

Это не существенный эффект, по сравнению с остальными.

>размерами баков не херится

Херится. Для сжигания 1кг (0.87л) кислорода нужно либо около 350 гр (437мл) керосина, либо 250гр (600мл) метана либо 125гр(1790мл) водорода, так как у жидкого водорода плотность 0.07кг/л. То есть турбонасос для водорода должен в 4 раза превосходить по производительности турбонасос для керосина и в три раза для метана.
Камера сгорания тоже должна быть больше. И сопло тоже.
И того:
1.баки для водорода должны быть в 4 раза больше керосиновых
1а. При этом они должны быть теплоизолированы.
1б. При этом наращивать в высоту до бесконечности их нельзя, ракета перевернётся, а в ширину тоже нельзя, ЖД габарит не позволяет. Ну то есть можно, только возить дорого будет.
2. Турбонасос должен быть мощнее.
3. Сопло и камера должны быть больше.
4. УИ на уровне моря не такой высокий, так как большое сопло даёт большое сопротивление атмосферного давления.

Поэтому у РД-0120 тяга была в 4 раза меньше, чем у РД-180, при этом РД-180 весил больше в 2.5 раза и был практически такого же размера.
Аноним 26/01/18 Птн 17:30:25  373751
Тесная двойная звезда состоящая из 2х ораньжевых может ли быть сопоставима по яркости с солнцем? Будут ли они стабильно выдавать свою светимость на 1 А.е. или ораньжевые так же нестабильны как и красные или их вращение будет сильно сказыватся на температуре?

Вопрос именно о температуре поверхности, а не о количестве свечей.
Аноним 26/01/18 Птн 18:09:05  373758
>>373751

Температура поверхности зависит от размера. От вращения температура поверхности не зависит.

Аноним 26/01/18 Птн 19:53:18  373781
>>373250 (OP)
>DLZ7X4rWkAc7c25.jpg
Меня в гугле забанили, кто этот дедушка и за какие заслуги его поместили в шапку? Нейросеть состарила Владимира Рыжкова?
Аноним 26/01/18 Птн 20:07:24  373785
>>373781
Риккардо Джаккони (итал. Riccardo Giacconi; род. 6 октября 1931, Генуя, Италия) — американский физик итальянского происхождения, лауреат Нобелевской премии по физике в 2002 г. Получил половину премии «за создание рентгеновской астрономии и изобретение рентгеновского телескопа».
Аноним 26/01/18 Птн 20:59:49  373795
>>373758
Размера чего?
И как ты будешь складывать 2 ораньжевых тельца, если они переодически 1.перекрывают друг друга. 2. переодически отдаляются\приближаются от планеты.
Вопрос в том критичны ли эти перепады для биоты в целом.
Аноним 26/01/18 Птн 21:57:30  373798
https://ru.wikipedia.org/wiki/Старт_(ракета-носитель)
Ну, как ни крути, шесть успешных запусков из семи. Наибольшая ПН - 350 кг. Неблохо, ящитаю. Потенциал имеется. На планетах с меньшей силой гравитации можно и побольше выводить.
Аноним 26/01/18 Птн 22:17:15  373807
>>373795
>Размера чего?

Звезды.

1.Не на долго.
2. Небесные тела вращаются вокруг общего центра масс, то есть если расстояние между звёздами значительно меньше расстояния до планеты, то всё нормально и жизнь не будет сильно отличаться от земной. Кроме того, Земля заселена различными организмами практически вся, а не только в зонах с наиболее благоприятным климатом.

>>373798
>Наибольшая ПН - 350 кг. Неблохо, ящитаю.

Смотря с чем сравнивать. Напомню, что баллистическая ракета дороже космической при той же энергетике.
Аноним 26/01/18 Птн 23:05:31  373818
>>373751
Оранжевые карлики стабильнее не только красных, но и солнцеподобных жёлтых карликов. В т.ч. и нашего Солнца. Если честно, то это самые козырные для жизни звёзды во Вселенной.
>>373807
Главный плюс "Старта" - его мобильность. И не просто мобильность, а мобильность на уровне "запуск из любых ебеней".
Ну, может и не из любых, тут уже нужно смотреть характеристики самого тягача - проедет ли он с ракетой по труднопроходимой местности вроде пустыни, заболоченных низин, полярных пустошей или мест, покрытых густой растительностью, вроде джунглей и тайги.
Аноним 27/01/18 Суб 01:32:08  373840
>>373818
>Главный плюс "Старта" - его мобильность.

И зачем это запускателю космических ракет?
Аноним 27/01/18 Суб 01:37:13  373841
>>373840
Можно фотографировать пуски на разном фоне, очевидно же.
Аноним 27/01/18 Суб 02:12:15  373847
>>373840
Чтобы было удобнее выводить на разные орбиты.
Аноним 27/01/18 Суб 02:27:12  373848
>>373847

Ты говоришь, как будто нужно много забрасывать. При такой нагрузке на НОО особо не приходится выбирать орбиту.
Аноним 27/01/18 Суб 14:00:07  373926
c4985923a8718d9[...].jpg (104Кб, 940x627)
На МКС будут делать бутерброды из говна астронавтов
https://geektimes.ru/post/297597/
Аноним 27/01/18 Суб 14:58:58  373938
15150537303352[[...].jpg (548Кб, 1920x1080)
15150537303300[[...].jpg (358Кб, 1920x1080)
Шалом, норко_маны.
Короче, взгляни на пики - есть овальнаядынькаоидная планета. Она имеет стабильное положение в зоне комфорта и сходные с земными насколько это возможно условия - на ней даже есть жизнь. Допустим, даже разумная человекоподобная. А как у этой человекоподобной жизни будут обстоять дела на такой планете? На это йпланете таки есть вода, пускай и магнитное поле (откуда-то). Но как такая форма планеты повлияет на гравитацию и расположении вод вообще? Какой местные господа видят эту планету при жизни?

Второй вопрос: есть планета на орбите чёрной дыры. Допустим, её орбита ультрастабильна и составляет идеальный круг вокруг центра массы ЧД и, допустим же, радиус окружности орбиты составляет 3 радиуса горизонта событий. И вот летит эта ЧД по космасу и тут из-за угла ей навстречу одинокая звезда вылетает. Ну ЧД к ней подкрадывается и начинает шмонать на массу и материю. При этом отжим материи происходит так, что планете хоть бы хны - допустим, потому что ЧД сосёт строго через полюс тонкой струйкой. Дальше такое дело: ЧД высасывает всю звезду и, соответственно, увеличивается сама. Вот тут уже я встаёт сам интересующий меня вопрос - как изменится орбита планеты при росте ЧД? Орбита останется неизменной и ЧД в итоге пожрёт планету? Или Орбита будет расти соразмерно увеличению ЧД? Если будет расти, она будет расти таким же кругом или заовалится?
Аноним 27/01/18 Суб 15:01:11  373939
Посоны, а тут есть кто из этих всяких ЦУПов или типа того, короче кто реально (а не в рамках лабы в шараге) занимается баллистическим обеспечением полётов, расчётами орбит/манёвров/сближений, вот это всё?
Чем сейчас обычно пользуются для высокоточных долгосрочных прогнозов, кроме численных расчётов? В опенсорце альтернатива SGP/SDP существует? Orekit есть какой-то, кто-нибудь пользовался?
Аноним 27/01/18 Суб 15:10:27  373942
>>373938
1. Ось вращения как проходит? Какова вытянутость, отношение большего радиуса к меньшему?

2.
> её орбита ультрастабильна
>из-за угла ей навстречу одинокая звезда
... и стабильности орбиты пизда.
Аноним 27/01/18 Суб 15:11:32  373944
>>373938
> дынькаоидная планета
У тебя на пике практически Хаумеа, но она не полноценная планета. что-то размером с Марс должно крутиться с совершенно пиздецовой скоростью, чтобы иметь такую форму.
Аноним 27/01/18 Суб 15:16:19  373945
>>373938
У тебя не планета. Критерий планеты - достаточная массивность для сферической формы. Мелкопланета формой с дыньку не удержит атмосферу.
Про ЧД этот правильно сказал >>373942
Аноним 27/01/18 Суб 15:16:31  373946
>>373938
Даже гипотетически гадать про твою дыню не хочется, все равно балагурство это. Но можно сказать, что фокальные удлинения будут по сути вести себя как гигантские горы, т. е. будут линиями водораздела. Водичка будет стекаться к экватору или блять где тут что? Ну в желобках будет скапливаться само собой, но сам вектор потока воды будет устремлен к области наименьшего сечения эллипсоида. То есть с полюсов будет течь оче много рек, довольно стремительных. Хотя если предположить, что эта форма вызвана центробежной, то полюса, ай впизду, хуита какая-то.
Сама по себе звезда, уплетаемая чд повлияет на орбиту и распидорасит к хуям эту орбиту. Никакой стабильности в этой вселенной предположить уже нельзя. Если взять совсем уж мысленный эксперимент, в котором чд массой в три солнечных вдруг прирастает на одну дополнительную, то планетке прямой путь в жерло шредера материи через какое-то количество витков. Стабилизируются орбиты на основе конкретных параметров. Если один из этих параметров резко меняется и сильно меняется, то за ним же уходит и стабильность. Она ведь заключена в балансе орбитальной центробежной и притяжения массивного объекта. Если массивность увеличивается, то для сохранения стабильности нужно увеличивать орбитальную скорость.
Аноним 27/01/18 Суб 15:38:52  373960
>>373938
>дынькаоидная планета
Как уже говорили, гравитация стремиться сжать всё к сферической форме. Если получилась такая хренота, как на твоём пике, то возможны два варианта:
1. Сила притяжения на ней сравнима с силой притяжения на Макемаке. Короче говоря - карлан, лишённый атмосферы.
2.Эта планета очень быстро вращается вокруг собственной оси. Но так как она, очевидно, не разорвана найух центробежными силами, следовательно им что-то противодействует. И скорее всего, это "что-то" - сила притяжения, причём настолько огромная, что не совсем понято, как твой шарик не стал газовым гигантом.

>составляет идеальный круг
В дальнейшем лучше не употребляй это словосочетание.
Аноним 27/01/18 Суб 15:39:45  373961
>>373960
>Макемаке
Пардон, конечно же Хаумеа.
Аноним 27/01/18 Суб 15:41:57  373962
>>373938
Если во втором сценарии заменить звезду на что-нибудь, не влияющее на орбиту планеты, например, стрелять в ЧД потоком частиц, то по мере увеличения массы, представь себе, планету начнёт затягивать в ЧД.
Аноним 27/01/18 Суб 16:13:51  373976
russell-crowe-m[...].jpg (340Кб, 1800x1200)
Будет ли гуманоид, прибывший с суперземли невероятно сильным и крепким относительно земной флоры? Или же по всем законам этот гуманоид будет очень маленьким по причине колоссальной силы тяжести, в которой он эволюционировал, даже если вообразить высокую плотность атмосферы на его родной планете?
Аноним 27/01/18 Суб 16:21:10  373981
>>373960
>Макемаке
Мэйкмэйк
ftfy
Аноним 27/01/18 Суб 16:27:31  373989
>>373981
Нет.
Аноним 27/01/18 Суб 17:19:56  374013
Есть ли альтернативы химическим ракетам для вывода на орбиту?
Аноним 27/01/18 Суб 17:22:57  374015
HIdCpwP2ASs.jpg (67Кб, 783x960)
>>374013
Кинетика. Но человек не прокачивал эту ветку технологий, так что запуск гигарогатками с Эвереста как единого всепланетарного пускового центра в вашей версии вселенной заблокирован.
Аноним 27/01/18 Суб 17:30:34  374017
>>374013
С земляшки в обозримом будущем - нет.
Вообще - петля лофстрома или космический лифт.
Зацени, в общем:
https://en.wikipedia.org/wiki/Non-rocket_spacelaunch

>>374015
Ты опять начинаешь со своими катапультами?
Аноним 27/01/18 Суб 18:31:49  374035
>>373976
>Или же по всем законам этот гуманоид будет очень маленьким по причине колоссальной силы тяжести
This, если гравитация вообще жопа, то это вообще может стать чем то похожем на насекомое с их спецификой тела, но в таком случае есть большие сомнения что оно сможет в разум. Впрочем если у планеты будет реальн толстая и достаточно плотная атмосфера чтоб обладать хорошей силой выталкивания, то может какой гуманоид и возникнет с человеко размерами. Но такой если покинет среду со своим йоба давлением то есть распидорасит, если он конечно не что то вроде плоских рыб из марианской впадине и т.п. то есть не поддерживающий в себе давления в противовес, но такой может и не суметь "ходить" в нашей атмосфере ибо ляжет пластом на Землю, но я хз, если не совсем "плоский червь" то сможет.

Либо его мышечные\ костные ткани должны состоять из чего - то прочнее чем типичные человеческие, так же нужна более продвинутая энерго система что бы все это питать пока оно пытается качать кровь в условиях нескольких жэ например. Есть сомнения что наша реальность поддерживает такой контент.
Аноним 27/01/18 Суб 18:56:13  374038
>>373537
Какой практический толк от гелия-3?
Аноним 27/01/18 Суб 20:43:45  374051
>>374038
Пока никакого, в фантазиях хотят термоядерные реакторы на нём делать.
Аноним 27/01/18 Суб 20:47:12  374052
>>374038
Можно обмазываться и дрочить.
Аноним 27/01/18 Суб 21:10:57  374061
15141295465030.jpg (642Кб, 1366x768)
>>373942
> 1. Ось вращения как проходит? Какова вытянутость, отношение большего радиуса к меньшему?
Вариант А: через самые длинные участки дынькаоида (жопки)
Вариант Б: через самые узкие участки дынькаоида
Вариант В: под углом так, чтобы ось вращения была под углом от 30 до 60 градусов относительно оси жопок дынькаоида
>>373944
Почему? Может внешние слои на определённом этапе выплеснули металлокаменную породу, да она и оказалось суперпрочной, да так и застыла, например.
>>373945>>373960
> Критерий планеты - достаточная массивность для сферической формы
Плутон, Луна, харон и прочие доказывают, что они планеты - они же круглые.
> Мелкопланета формой с дыньку не удержит атмосферу.
Но почему? Ведь дело в плотности планеты. Именно она отвечает за массу. Значит, и воздух будет держаться. Если я не прав, то в чём?
Аноним 27/01/18 Суб 21:23:22  374063
>>374061
>Если я не прав, то в чём?
Космоаксиома: если космическое тело обладает массой, достаточной для удержания плотной атмосферы, то данное космическое тело обладает формой близкой к шарообразной.
Аноним 27/01/18 Суб 23:09:52  374090
>>374061
>Плутон, Луна, харон и прочие доказывают, что они планеты - они же круглые.
Круглость — необходимое условие, но не достаточное (на самом деле, даже не то, чтобы необходимое, если тело неправильной формы расчистит себе орбиту вокруг звезды, то придётся считать его планетой, просто так не происходит. Как и тел неправильной формы, способных удержать атмосферу.)
Аноним 27/01/18 Суб 23:31:23  374096
>>374061
> Может внешние слои на определённом этапе выплеснули металлокаменную породу, да она и оказалось суперпрочной, да так и застыла, например.
С этим тебе в сайфач. ИРЛ земная кора даже под Гималаями емнип прогибается, а всё, где этого не будет, не попадает под один из необходимых критериев планеты: поддержание сферической формы с помощью гидростатического равновесия, посколько требует слишком маленькую для этого силу тяжести.
> Ведь дело в плотности планеты. Именно она отвечает за массу. Значит, и воздух будет держаться. Если я не прав, то в чём?
В том, что или форма дыньки, или достаточная масса. И того и того не бывает.
Аноним 27/01/18 Суб 23:49:47  374099
>>374035
То есть те былины о десятиметровых богатырях вполне могут быть правдой, если они сами забафали себя генной инженерией, но "природная эволюция" сверхсильных существ не завозит?
Аноним 28/01/18 Вск 01:32:17  374126
Поясните, не понимаю: почему на зондах и АМС без солнечных батарей приходится ставить РИТЭГи на дорогом изотопе плутония? Причем там называют какие-то страшные суммы, а очередь на несколько кило этого изотопа плутония ждут по 5-10 лет и даже были вынуждены в своё время унижаться и покупать за оверпрайс у России. А сейчас нету плутония - нету дальнего космоса. Многомиллиардные научные проекты идут по пизде.

Но при этом для обогрева полярников и спутников Земли РИТЭГи собирали(ют) на основе деления какого-то мусора. Стоит это на порядок или даже несколько дешевле.

Да, я слышал про энергомассовое совершенство, но википедия говорит, что, например, стронций-90 выделяет почти столько же энергии (0.46 ватт против 0.54 у плутония-238). Период полураспада 29 лет, что за глаза и за уши. Да, он фонит куда сильнее, но не похуй ли? Почему всякие разведывательные спутники могут вращаться на стронции, а условный кьюриосити не может? Слишком много нежного оборудования, что экранирование всего и вся не получится? Какие-то научные инструменты не могут работать при таком фоне? Другие причины?

Почему Луна-25 и Луна-26 по планам будут иметь на борту РИТЭГи для обогрева (на каком топливе?), но при этом электроэнергию они будут брать через солнечные панели?
Аноним 28/01/18 Вск 06:44:55  374152
>>374096
>В том, что или форма дыньки, или достаточная масса. И того и того не бывает.
А что фундаментально мешает вращаться планете массой с Землю например со скоростью не 300 метров в секунду, а 3 км в секунду, и тем самым растянуться по экватору?
Аноним 28/01/18 Вск 07:39:09  374154
6xBomGd2BjA.jpg (79Кб, 1280x822)
так абажжи ёбана
а если мы возмём сферическую планету из плотного и прочно по планетарным меркам материала в поверхности, а затем с двух концов - например на полюсах - накидаем много лёгкого и объёмного материала типа пемзы или аэрогель. тогда получится дыняоида и не будет проблемы прогиба коры
Аноним 28/01/18 Вск 08:50:22  374157
14402831932710.jpg (50Кб, 640x446)
>>374154
Ну пемза имеет свой предел прочности, а аэрогель надо ещё где-то достать.
Аноним 28/01/18 Вск 08:57:53  374158
>>374126
Я скажу честно, что не знаю.
Но 29 лет это по космическим меркам с гулькин клюв. Пока долетишь до нептуна четверть стронция уже разлетится а следствие и электричества будет нема. Ну и четверть массы выстреливающие с релятивистской скоростью это тебе не хухры-мыхры.
Аноним 28/01/18 Вск 09:13:54  374161
>>374152
Приливные силы звезды мешают. Если достаточно далеко от звезды, то может и можно.
Аноним 28/01/18 Вск 09:36:17  374163
>>374152
> А что фундаментально мешает вращаться планете массой с Землю например со скоростью не 300 метров в секунду, а 3 км в секунду, и тем самым растянуться по экватору?
То, что хрен ты такой угловой момент наберёшь.
>>374154
> а затем с двух концов - например на полюсах - накидаем много лёгкого и объёмного материала типа пемзы или аэрогель. тогда получится дыняоида и не будет проблемы прогиба коры
И аэрогель, естественно, не будет сжиматься под своим весом.
Аноним 28/01/18 Вск 10:00:56  374168
>>374163
будет, но только ты сравни плотность какого-нибудь гранита, базальта или хотя бы бетона с аэрогелем. что тяжелее - один кубокилометр чего-то из этого или один кубокилометр последнего?
Аноним 28/01/18 Вск 10:03:59  374169
>>374168
Один километр аэрогеля начнёт складываться тупо под своим весом, и чтобы его получить, тебе придётся навалить аэрогеля столько, что пористым он будет только у самой вершины.
Аноним 28/01/18 Вск 10:52:07  374176
Можно ли сварить пельмешки на МКС?
Аноним 28/01/18 Вск 11:32:31  374186
>>374176
Да.
1) Берём воду и пельмешки
2) Берём подковообразную тару
3) Наливаем воду с расчётом на увеличание объёма воды. которая превратится в пар выкипанием прежде, чем истечёт расчётное время приготовления пельмешек.
4) Подставляем воду с солью и перцем с погружёнными пельмешками под лазеры/подключаем к системе охлаждения
5) Всё по науке - кипятим воду ровно столько, сколько нужно
6) Едим пельмешки и запиваем его бульоном
7) ???????
8) DOVOLbNO URCHIM
Аноним 28/01/18 Вск 12:08:06  374195
>>374186
Не тупи, в микроволновку поставь просто.
Аноним 28/01/18 Вск 12:52:06  374210
>>374017
>>374015

Не, я не про шизофреничные проекты, а про ракеты с другими типами двигателей. Ну там ярд, термальные и прочее, это вообще развивается?

Кстати, что там насчет совокупления Роскосого и Росатого с целью создания мощного ярд? Какие у него задачи?
Аноним 28/01/18 Вск 12:57:48  374212
>>374186
Лол
>берём тару с клапаном
>заполняем водой доверху
>кидаем туда пельмешки, соль, перец горошком, душистый перец, кориандр, лавровый лист
>суём туда кипятильник, спизженный у сторожа на Байконуре
>пар вырывается через клапан, пельмешки варятся
>скармливаем это дерьмо пиндосам и едим великолепные манты, которые готовились в это время на пару
Аноним 28/01/18 Вск 13:16:44  374217
3a0e556007c2d31[...].gif (8884Кб, 286x278)
>>374212
Как говорил Жан Фреско, слышь, ты ебало-то завали. Вопрос был про пельмени, а не манты. Каково техзадание, таково и инженерное решение.

Кстати, у меня вопрос.
Я лечу внутри космического корабля. Причём корабль этот - классической ракета на реактивной тяге из фильмов о будущем времён 50х+ годов Ну вы поняли. Так вот, на какой скорости должна лететь ракета, чтобы я начал чувствовать гравитацию за счёт давления тягой вниз, к соплу? Я чувствую в своём собственном вопросе подвох, но пока его не вижу
Аноним 28/01/18 Вск 13:22:37  374218
>>374217
Ты бля сначала хоть один урок физики в школе посети, а потом будешь про ебала вякать. Пельмени тоже готовы, но манты лучше получатся в таких условиях.

Похуй какая у тебя скорость, тебе ускорение нужно. Угадаешь какое? 9,81 метр в секунду в секунду.
Аноним 28/01/18 Вск 13:29:32  374220
>>374186
>>374212
Вы забыли, что конвекции в невесомости нету, потому надо как минимум посуда с автомешалкой.
Аноним 28/01/18 Вск 13:35:35  374223
>>374220
Для это есть химотел с центрифугой или магнитной мешалкой в склянках.
Аноним 28/01/18 Вск 13:39:00  374226
>>374217
внезапно, с ускорением в несколько метров в секунду в квадрате
Аноним 28/01/18 Вск 14:04:03  374229
>>374220
>конвекции в невесомости нету
Нипонил. Эт почему?
Аноним 28/01/18 Вск 14:11:59  374234
15166293847330.jpg (31Кб, 429x472)
Чё там с масштабами чёрных дыр?
Вроде, есть чёрные дыры, у которых такие размеры, что вся солнечная система для них, как Плутон для Солнца.
Где пределы размеров ЧД?
Если с набором определённой массы звезда коллапсирует в ЧД, то в каких случая ЧД коллапсирует до вызрывообразного и экстрабыстрого рассасывания по квантовым туннелям (или что там Хокинг предположил)?
Может ли быть, что вся хронология жизни на вселенной - это просто момент цыкла вдоха-выдоха ЧД, состоящей из всей материи вселенной? Типа
1. Большой взрыв
2. Зарождение туманностей
3. Зарождение звёзд
4. Зарождение протоплазменных дисков
5. Зарождение планет
6. Первые смерти старейших звёзд и первые чёрные дыры
7. Все звёзд умирают, превращаясь в ЧД или нежзнеспособные отбросы звездоэволюции
8. Во вселенной остаются только чёрные дыры, поглащающие остатки планет и материи.
9. Все ЧД неотвратимо тянутся друг к другу образуя такие же планетарные системы, как в СС - где есть звезда, большие планеты, малые планеты, луны-спутники, астероиды, метеориты и камушки-мусор, только ничего не светится и не излучает ничего, ибо вся вселенная состоит из чёрных дыр, и все чёрные дыры поменьше вращаются вокруг чёрных дыр побольше.
10. Все чёрные дыры постепенно сливаются в одну всевселенскую чёрную дыру
11. Под своей тяжестью эта ЧД коллапсирует квантовыми рассосочными туннелями во все стороны простарнства и времени сразу
12. Вернуться в пункт 1.

Таким образом цикл жизни вселенной похож на прекрасную бабочку: сперва фаза личинки - вся вселенная забита звёздами и протопланетными дисками, затем фаза - куколки - падения числа звёзд и рост числа ЧД, и наконец фаза бабочки - все вселенная наполнена чёрными драми и ничего кроме них во вселенной нет и все, даже мельчайшие пылинки - мини-чёрные дыры. В конце "жизненного цикла" вселенная чёрнодырной бабочки откладывает яйцо - всевселенскую чёрную дыру, которая под своим весом коллапсирует и снова повторяется цикл.
Аноним 28/01/18 Вск 14:13:00  374235
>>374229
http://posmotre.li/Забыли_про_гравитацию
Аноним 28/01/18 Вск 14:22:11  374236
>>374235
Хотя вот эта статья, пожалуй, даже лучше http://posmotre.li/Какая_такая_конвекция%3F
Аноним 28/01/18 Вск 14:23:55  374238
>>374234
Трабл в том, что очень быстро материя улетает за горизонт событий друг от друга из-за расширения вселенной и в принципе не может притянуться друг к другу.
Аноним 28/01/18 Вск 14:29:00  374239
>>374210
>шизофреничные проекты
Щас обидно было.
>ракеты с другими типами двигателей. Ну там ярд, термальные и прочее, это вообще развивается?
Пробовали лазоры, но атмосфера быстро кончается и мешает с ростом расстояния, так что всерьез не рассматривали. Иные способы не обладают тяговым совершенством, тот же же ЯРД обладает низкой тяговооруженностью и не может поднять даже сам себя.
Рассматривали его в качестве третьей ступени на Сатурн, но в случае фейла при старте пришлось бы чистить окиян от изотопов, чего никому не охота, кроме пизданутых узкоглазых, которым насрать и которые даже дельфинов убивают, пидоры ебанутые
Аноним 28/01/18 Вск 14:30:02  374240
>>374234
Чем больше чд, тем дольше она рассасываться будет излучением Хокинга
Для чд звездной массы нужны триллионы лет, для галактической вообще пиздец, а для твоей вселенской дыры, которая кстати невозможна из-за расширения вселенной, почти бесконечный промежуток времени нужен
Аноним 28/01/18 Вск 14:32:12  374242
>>374240
У черной дыры же время иначе идет.

>>374234
>Все ЧД неотвратимо тянутся друг к другу
Нет, пока что все отталкивается и разлетается, этот верно сказал >>374238
Аноним 28/01/18 Вск 14:34:26  374244
>>374235
>>374236
Это что, отечественный дешманский ТВТропс?
Аноним 28/01/18 Вск 14:39:33  374247
>>374242
>>374238
Возможно, что время/гравитация - это вторая скоростная константа, как света. Только намного больше. например, при обычной ЧД свет уже не может покинуть, а время/гравитация - могут, но при ЧД галактических размеров время/гравитация уже начинают тяготеть обратно, и как бы далеко не успели распространиться, с появлением первой подобной ЧД, время/гравитация как тягучая масса, как яичный белок, натекают стекаться обратно, нивелируя тем самым расширение вселенной. Вселенная начинает сужаться, ЧД беспрепятственно сливаются, всё быстрее поглощая всё, включая время/гравитацию и в конце концов, когда последняя... крупинка? времени/гравитации вернётся во лоновсевселенской ЧД, тогда уже и Большой Взрыв.
Аноним 28/01/18 Вск 14:41:14  374248
15162193948430.jpg (204Кб, 1893x888)
>>374244
Изначально да, сейчас это уже что-то самостоятельно и что-то типа лурка, но более википедичное, строго ориентированное на творческий контент - ТВ, игры, книги и т.д.
Аноним 28/01/18 Вск 14:44:31  374249
>>374247
Таких официальных теорий нет. Предполагается, что вселенную распидорасит в ничто.
Аноним 28/01/18 Вск 14:46:02  374250
>>374249
> Таких официальных теорий нет. Предполагается, что вселенную распидорасит в ничто.
Хуйня какая-то. Мне не нравится. У меня теория лучше.
Аноним 28/01/18 Вск 14:48:27  374251
>>374163
>То, что хрен ты такой угловой момент наберёшь.
Но... Земляшка когда в неё въебалась Тея приобрела как раз подобную скорость http://jhooty.2291.ru/index.php/201-2/221-2/222-2/ Если бы она тогда умудрилась сожрать все говна не выводя ничего не орбиту или отшвырнуть их со второй космической, ну или вывести на орбиту на порядок меньше массы, сомневаюсь, что она бы так сильно замедлила своё вращение к нашему времени. Но замедлила бы всё равно ибо приливные силы Солнца никуда не делись.
Аноним 28/01/18 Вск 14:48:49  374252
>>374250
>Мне не нравится
Соболезную.
Аноним 28/01/18 Вск 14:58:45  374254
147610709513272[...].gif (8029Кб, 380x274)
Так, стоп, а откуда берётся протопланетный диск, из которого появляются и звезда и планеты, и спутники?
Вот та хуйня, из которой взялась солнечная система из чего получилась? Почему именно здесь и именно в таком количестве?
Аноним 28/01/18 Вск 15:11:20  374260
>>374254
Из межзвёздного газа, из плотных туманностей (молекулярных облаков). А они получаются из более старых звёзд, которые умерли, их тела превратились в траву, а антилопы едят траву.
Аноним 28/01/18 Вск 15:26:24  374265
>>374254
Из того же газа, что и сама звезда.
Аноним 28/01/18 Вск 15:45:22  374270
>>374260
>>374265
Но откуда у нас тут взялось столько газа? Почем именно столько, а не больше и не меньше? Что здесь пролетало и что ёбнуло, что столько материи раздристалось в пределах СС? Есть способы как-то просканировать это всё, чтобы узнать?
Аноним 28/01/18 Вск 15:53:56  374271
EarthAtmosphere[...].jpg (69Кб, 685x599)
Дано: я ни с того ни с сего перемщаюсь на орбиту МКС без начальной скорость, но с читом не возможность дышать в вакууме и неповреждаемость от радиации.
Вопрос 1: Через сколько я упаду на Землю?
Вопрос 2: Как правильно - и возможно ли - падать на Землю так, чтобы нагревание трение об воздух компенсировалось заморозками в атмосфере? Если у меня была возможность замедляться пару раз, где бы мне стоило замедляться, чтобы испытывать обморожение максимально мало?
Аноним 28/01/18 Вск 15:55:02  374272
>>374271
> на орбиту МКС без начальной скоростИ, но с читом нА возможность дышать в вакууме
Аноним 28/01/18 Вск 16:09:42  374273
>>374270
>Но откуда у нас тут взялось столько газа?
Ты явно не понимаешь, что такое астрономические масштабы.
>Почем именно столько, а не больше и не меньше?
Сколько получилось, столько и получилось.
>Что здесь пролетало и что ёбнуло, что столько материи раздристалось в пределах СС?
Скорей всего ничего не пролетало, просто поблизости ебнула сверхновая которая взбурлила это локальное облако газа.
Аноним 28/01/18 Вск 16:25:45  374275
>>374273
> Ты явно не понимаешь, что такое астрономические масштабы.
Прекрасно понимаю. А ещё понимаю, что для того, чтобы что-то где-то появилось, например, материя для того, чтобы у нас тут хватило и на солнце и на планеты, надо чтобы откуда-то взялось. Причём сразу и много, в противном случае, это всё осталось бы разряженным межзвёздным газом и не стало бы скапливаться, а осталось бы рандомным вихрями малоплотной межзвёздной пылевой тучки, то тут, то там равнозначно получающей и теряющие равны порции молекул и частиц
Аноним 28/01/18 Вск 17:05:19  374286
>>374275
> ещё понимаю, что для того, чтобы что-то где-то появилось, надо чтобы откуда-то взялось.

Если ты про "откуда взялось столько газа?". То ответ в том, что самое доступное во что согласно физическим законам могли завязаться частицы появившиеся после Большого взрыва были: вещества водород и гелий, а потом уже все начало закручиваться в YOBA туманности, после в Галактики и т.д.

Если ты про Большой взрыв, то есть:

Маня предположения о том что он либо взялся с НИХУЯ из самого настоящего НИХУЯ. Либо как вариант само начало Большого взрыва вертело на сингулярности собственные же законы физики которые и возникли в пространстве времени порожденном Большим взрывом.

Либо же, что более логично - флуктуация Вселенной само поддерживает себя "вечно", обеспечивая условия для возникновения Большого взрыва вновь и вновь. Ну или таки не обеспечивая, а сам Большой Взрыв это случайный элемент существующего, но тут уже совсем наркомания Но на вопрос почему такая флуктуация существует всегда, ответа все равно нет. Возможно тут сами мнения о существующем и не существующем, могут не иметь смыслов, хз.
Аноним 28/01/18 Вск 17:32:00  374293
>>374234
>Где пределы размеров ЧД?

Радиус ЧД растёт ЛИНЕЙНО с ростом её массы.
Аноним 28/01/18 Вск 17:37:45  374295
>>373294
>Хокинг.Жэпэг

Тем временем ВБыдлятне
Аноним 28/01/18 Вск 21:33:47  374361
>>373250 (OP)
Почему кто-то считает, что неизвестно, конечна ли наша Вселенная или безгранична ? Потому что если Вселенная имеет конец, то как это по их мнению должно выглядеть ? Кирпичная стена и шлагбаум ?

Очевидно, что космос беграничен. Таким образом, становится доказанным существование других миров, в том числе обитаемых, потому что если космос безграничен, то и количество возможностей для воспроизводства жизни (причем не только земного типа) также безграничны, вопрос лишь в расстоянии до них. Да в принципе и вопрос о Б-ге в таком случае тоже можно считать доказанным, т.к. в одном из таких миров уровень развития несопоставимо превышает земной и в любом случае трансформировался либо в чистую энергию, обладающую сознанием, либо что-то еще, что в любом случае может влиять на любые процессы вселенной.
Аноним 28/01/18 Вск 21:42:57  374362
>>374361
Ещё один. В шапку уже надо добавить, что краткая история времени очень рекомендуется к прочтению. Короче смотри: представь, что ты 2d ее, аниме. Ты такой ищешь в своей вселенной конец, а его нет. Но возвращаешься в одну точку. Почему? Потому что такая вселенная это 3d шар. Она конечна, но не имеет границ.
Так и у нас, мы 3д, а вселенная 4д шар. Вообще тут 11 измерений, но это другая история
Аноним 28/01/18 Вск 21:43:00  374363
>>374361
>Потому что если Вселенная имеет конец, то как это по их мнению должно выглядеть ? Кирпичная стена и шлагбаум ?
>Очевидно, что космос беграничен.
Ты типа сейчас обосновал свою теорию тем, что никто не знает, как может выглядеть граница космоса, если таковая существует, поэтому она безгранична? Ты там совсем охуел что ли?
Аноним 28/01/18 Вск 22:13:31  374367
>>374361
Школьник срывает покровы
Аноним 28/01/18 Вск 22:33:52  374376
>>374295
Всегда печёт с этой пикчи. Очень печёт.

>>374361
> как это по их мнению должно выглядеть ?
"Выглядение" возможно только в этой вселенной.Она либо замкнута в себя, либо нет. Тогда ты увидишь ту же вселенную, либо ничего.
>Очевидно, что космос беграничен.
Кому очевидно? Почему? Нет признанной теории считающей так. Космос ограничен, масса вселенной примерно подсчитана и составляет 1053 килограмм.
Аноним 28/01/18 Вск 22:50:18  374381
>>374361
>Если Вселенная имеет конец, то как это по их мнению должно выглядеть ?
Если Вселенная - четырехмерная гиперсфера, то конец вселенной будет выражаться в том, что при движении в один конец ты вернешься в то место, откуда начинал.

Аноним 28/01/18 Вск 23:01:49  374383
>>374381
>четырехмерная гиперсфера
Но она трехмерная
Все в этом мире трехмерное
Аноним 28/01/18 Вск 23:42:20  374389
>>374235
Ну блять, между выводами "конвекции нету" и "конвекцию надо стимулировать" все же я разница на мой взгляд. Ладно, вопрос закрыт.
Аноним 28/01/18 Вск 23:46:41  374390
>>374389
Её и нету, вместе с силой тяжести пропадает и любая конвекция, что в воздухе что в воде.
Аноним 28/01/18 Вск 23:47:53  374391
>>374234
Ты описал "большое сжатие". Современные данные пока явно не в пользу оного. Из трех сценариев БП этот самый маловероятный. Но насчет космологической константы уверенности пока не то, чтобы много.
Аноним 29/01/18 Пнд 00:27:48  374393
1353352730300.jpg (209Кб, 900x1360)
>>374271
>Через сколько я упаду на Землю?
t=(2s/g)0.5
До 40 километров сопротивлением атмосферы можно пренебречь, а дальше все равно пиздарики.
>и возможно ли
Нет. Вот смотри. Что такое температура? Температура - усредненный параметр кинетической энергии частиц окружающей среды, если попроще. Средняя скорость молекул в воздухе при н. у. около 500 м/с. Понятно, что они движутся хаотично и векторы столкновения с телом в большинстве случаев находятся под углами, что снижает эффективность передачи энергии, но в нашем случае для объяснения не будем заморачиваться с векторами и просто для наглядности примем, что температура воздуха 20 градусов выше нуля соответствует средней скорости молекул 500 м/с. 20 градусов - комфортно и удобно. А вот теперь возьмем реактивную авиацию. На сверхзвуке уже существуют проблемы с термостойкостью материалов, а это всего лишь 400 м/с добавочного вектора скорости. То есть даже меньше стандартных 500, да еще и летают они высоко, где атмосфера разряженней и температуры ниже. Но смысл в том, что кинетическая энергия растет с квадратом скорости. и между 500м/с и 900м/с разница не в 1,8 раз, а в 9002/5002, то есть квадрат линейной разницы, 3,24. И вот по этой причине американский йоба скоростной разведчик SR-71, бороздивший просторы разряженной стратосферы на трех махах, разогревался до 400 градусов. Хотя там очень мало воздуха и воздух этот холодный. А ведь это даже скорость меньше километра в секунду. На самом деле аэродинамический нагрев - тема куда более сложная, завязанная на параметрах вязкости среды и обтекаемости+парусности тела, да и я в ней, будем честны, не шарю практически вовсе, чтобы рассусоливать. Модель сложения скоростей очень грубая, зачастую далекая от реальности, но для общего представления схемы теплообмена, думаю, понятная.
Так вот, по вышенаписанной формуле, низвергаясь с высоты мкс, падая в поле тяготения Земли без сопротивления среды, тело к стратосфере достигнет скорости 2,5 км/с. Пусть разряженность воздуха будет предельно велика, а температура среды низка, тем не менее кинетическая энергия частиц среды будет превышать показатели привычных на уровне моря в 30+ раз. То есть даже абсолютно покоящиеся относительно друг друга частицы в среде, пресловутый абсолютный ноль, на таких скоростях криогенного эффекта оказывать на движущееся тело не будут, просто будет лишь слегка прохладней. Ну, скажем так, существование атмосферы при абсолютном нуле сомнительно, просто для наглядности. Тут нет уже смысла говорить об охлаждении средой. По сути принцип вычитания в тепловом балансе просто не будет иметь места. И само собой в плотности слоев тропосферы, тело, падавшее из космоса и пусть даже замедленное средой, просто сгорит нахуй в пепел, словно основная часть челябинского метеорита. Хотя метеорит бахнул гораздо выше тропосферы. В случае с человеком в скафандре все случится и подавно выше, а от теплового удара он помрет совсем высоко.
Аноним 29/01/18 Пнд 00:38:42  374394
>>374275
>Прекрасно понимаю
Всегда лоллирую с подобных заявлений. Перефразирую их для себя так: "Я достаточно насмотрелся научпопа, в котором туманности такие бдыщь и вжух, поэтому теперь достаточно компетентен, чтобы подвергать сомнению официальные научные теории." Нет, в слепую веру официальную линию науки превращать нельзя. Здравый скепсис - наше научное все. Ну так ты матаппарат для начала примени, хоть самый грубый и простой, чтобы подкрепить свои сомнения самым универсальным и общепризнанным техническим языком, а не выдавай вот такие сентенции
>чтобы у нас тут хватило и на солнце и на планеты
Аноним 29/01/18 Пнд 00:42:47  374396
>>374361
>вопрос о Б-ге в таком случае тоже можно считать доказанным
Ну на такое нельзя не вставить про критерий Поппера, хоть нынче про него в спейсаче и моветон упоминать, рассчитывая, что совсем уж дремучие все же в других местах сначала ума набираются.
Аноним 29/01/18 Пнд 00:47:38  374397
convectio.jpg (209Кб, 1425x387)
>>374390
>пропадает и любая конвекция
Бля, чувак, ну прекрати. Я не хочу буквоедствовать, но ты вынуждаешь. Я понял, я не спорю. Просто считаю, что "конвекции нет" звучит слишком скороспело.
Аноним 29/01/18 Пнд 00:50:40  374398
>>374271

Падая с высоты МКС успеешь разгонаться до скорости примерно 2500м/с до вхождения в плотные слои атмосферы, это 6 махов. Пизда короче.
Аноним 29/01/18 Пнд 00:52:59  374399
Если спутник находится на полярной орбите, вращается ли она вместе с планетой? Т.е., если смотреть со стороны Солнца, орбита будет всё время поворачиватся или оставаться под одним углом?
Аноним 29/01/18 Пнд 00:53:11  374400
>>374393
Я всё таки не совсем понял, сколько я буду падать времени, через сколько секунд падению (и на какой высоте) мне будет становиться жарко, на какой - конкретно припекать, а где - я уже сгорю нахуй.
И да, - раз уж это тупых вопросов тред - а если обмазатьсяобвеситься дюжиной огнетушителей всех мастей и выпрыскивать на себя пену, которая охлаждает меня за счёт испарения с забором лишней тепловой энергии?
Аноним 29/01/18 Пнд 00:54:57  374401
>>374013
Да, ядерные заряды под днищем.
Аноним 29/01/18 Пнд 01:01:18  374402
>>374400
>Я всё таки не совсем понял, сколько я буду падать времени
Ну блять, ну формула же. Совсем примитив. Расстояние - высота орбиты. g - ускорение свободного падение, 10м/с2 грубо. (400000х2/10)0,5 итого 282 секунды, если полностью пренебречь сопротивлением среды. Но пренебречь нельзя, а чтобы не пренебречь, нужно учесть довольно дохуя параметров из термодинамики, аэродинамики, даже небесной механики. Никто тебе этажи формул тут рассчитывать не будет. За это люди в цупах деньги получают. Но грубо можешь принять 5 минут, если взять какой-нибудь гипотетический обтекаемый объект из несгораниума.
>а если обмазатьсяобвеситься дюжиной огнетушителей всех мастей и выпрыскивать на себя пену
Рано или поздно, с учетом всех здравых мыслей и оптимизации в традициях лучших идей омского КБ, придем к жаропрочной капсуле. И вот в капсуле уже можно, даже проще, чем с орбиты, потому что скорости гораздо ниже.
Аноним 29/01/18 Пнд 01:09:24  374404
>>374401
Кстати, сейчас же ядерные бомбы стали "почище" бомб времен Ориона, не? Или все равно слишком мудацкая идея?
Аноним 29/01/18 Пнд 01:15:11  374405
>>374404
>Кстати, сейчас же ядерные бомбы стали "почище" бомб времен Ориона, не? Или все равно слишком мудацкая идея?

Примерно такие же остались. Можно взрывать не бомбы, выбрасывать плазму, стягивать её магнитным полем в критическое состояние, получать взрыв и так вжжжжжжжжжж по сто взрывов в секунду, а плиту опорную на пружинках закрепить.
Аноним 29/01/18 Пнд 03:45:27  374410
Вот есть обычные звёзды с термоядерным синтезом, горят себе и норм. Но хули светятся да и вообще существуют различные карлики и нейтронные звёзды, в которых нет никаких термоядерных реакций?
Аноним 29/01/18 Пнд 06:13:51  374413
>>374410
Они нагрелись в процессе звёздной эволюции\просто гравитационного сжатия. Вот и светят. Остывать будут очень долго. Десятки миллиардов лет, некоторые объекты вроде нейтронных звёзд возможно квинтиллионы лет если не больше.
Аноним 29/01/18 Пнд 08:15:49  374419
>>374399
Если просто: не вращается, со стороны Солнца будет все время поворачиваться.

Если посложнее: существует еще такая штука как прецессия, из-за которой орбиты спутников медленно поворачиваются вокруг Земли в направлении, противоположном направлению движения по орбите.

Соответственно, можно запустить спутник на околополярную ретроградную орбиту с такими характеристиками, что евоную орбиту начнет крутить на восток на 1 градус в день, и так как Земля вокруг Солнца тоже движется на 1 градус в день, то со стороны Солнца будет казаться, что орбита спутника не поворачивается. Профит!
Аноним 29/01/18 Пнд 09:19:29  374423
Не такой уж Ариан и надёжный, так что мистер Вебб может и потерпит крах.
Аноним 29/01/18 Пнд 09:52:07  374427
>>374397
> Просто считаю, что "конвекции нет" звучит слишком скороспело.
Ну у тебя же даже на скриншоте написано "в поле тяготения". Без него - только принудительная, что и было сказано в изначальном комментарии.
Аноним 29/01/18 Пнд 10:09:27  374428
>>374423
Скорее нортропы налажают и скоп подохнет сам по себе. Вспомни про знатный обосрамс с зеркалом Хаббла. Репорт.
Аноним 29/01/18 Пнд 10:10:57  374429
Правда что на МКС воняет шопиздец?
Аноним 29/01/18 Пнд 10:18:27  374430
>>374429
На МКС дует шопиздец
Аноним 29/01/18 Пнд 10:43:12  374434
Usa-Sky-Stone-D[...].jpg (267Кб, 960x720)
Взгляни на пикрелейтед.
У меня в голове такой образ возник: есть планета, представляющая из себя сферу, близкую к идеальному шару. Поверхность её представляет из себя целиком и полностью каменную пустыню типа как здесь, где куда ни кинь взор - всё ровно и одинаково - каменная равнина от тебя до самого горизонта. Ну, может, где-то есть холмы и даже горы, где-то овраги, но в целом всё ровно и одинаково. Но стоит заглянуть в щель а там батя сидит на карчиках между черепицами растрескавшейся поверхности, а там вода и растения и жизни. Ну то есть, естественно, щели эти в масштабы планеты представляют из себя расщелины глубиной в сотни или десятки километров, а шириной всего лишь в сотни метров - то есть они выглядят именно как щель, а не как пересохшее русло огромной реки.
Так вот, хотелось бы узнать, что вы думаете о жизни на такой планете. О погоде, о климате и т.д. для упрощения эта планета - Земля, только заменён ландшафт с того, что нам известно на вот такое описанное мной.
Аноним 29/01/18 Пнд 11:12:01  374441
>>374434
Подозреваю, что их быстро засыпет.
Аноним 29/01/18 Пнд 11:18:31  374442
>>374402
> И вот в капсуле уже можно, даже проще, чем с орбиты, потому что скорости гораздо ниже.
NANI?
Это ещё почему? Больше масса - больше влияния жэ - больше ускорения - более быстро наступает перегрев
Аноним 29/01/18 Пнд 11:21:06  374443
>>374427
>что и было сказано в изначальном комментарии
Где?
Аноним 29/01/18 Пнд 11:29:52  374444
>>374442
>Больше масса - больше влияния жэ
Нет, так это не работает. Ускорение свободного падения более менее постоянно для тел массой до миллиарда тонн, да и дальше тоже на самом деле. Дальше уже может как-то взаимное притяжение играть небольшую роль. У тебя формула силы притяжения какова? Произведение массы двух тел и гравитационной постоянной, деленное на квадрат расстояния. И да. Тело в десять раз тяжелее будет притягиваться Землей с силой в десять раз большей. Но! Так как масса тела тоже в десять раз больше, то по классике второго закона Ньютона ускорение от притяжения планетой будет одно. Именно поэтому в вакуумных трубках и перо, и камень ускоряются притяжением идентично. Увеличить ускорение от притяжения можно, когда помимо земного притяжения и само тело будет притягивать Землю. Очевидно, что разница в притяжении Земли что космонавтом в скафандре, что целым модулем, будет ничтожна мала в обоих случаях и ими можно пренебречь, а разница в ускорении будет плавать в районе двадцатых знаков после запятой.
Аноним 29/01/18 Пнд 11:34:30  374445
Цвет ионника зависит от газа?
Аноним 29/01/18 Пнд 11:56:54  374449
>>374429
нет, там стоят фильтры и различного рода установки по очистке воздуха
Аноним 29/01/18 Пнд 12:05:54  374451
>>374442
Ловите семиклассника
mg=ma, m сокращется
Аноним 29/01/18 Пнд 12:16:07  374453
>>374442
платина
Что быстрее упадет на Землю: яблоко, сброшенное с крыши небоскреба Москва-Сити или 16-кг гиря оттуда же на крышу проезжающего корте
Аноним 29/01/18 Пнд 12:25:02  374455
>>374434

Как в эти щели попадает вода? Почему эти щели не засыпало?
Аноним 29/01/18 Пнд 12:34:50  374457
15170818823510[[...].jpg (950Кб, 1600x1600)
На пикрелейтеде радиус самой чёрной дыры или уже горизонта событий?
Аноним 29/01/18 Пнд 12:42:23  374459
>>374458
>Тред тупых вопросов
А что не так? Лучше здесь, чем где-то еще
Аноним 29/01/18 Пнд 12:43:56  374460
>>374457
А это не одно и то же?
Аноним 29/01/18 Пнд 12:44:09  374461
>>374455
> Как в эти щели попадает вода? Почему эти щели не засыпало?
Например, вода не попадает туда, а априори есть там. То есть, уровень моря тут именно тут, на дне расщелин. И все воды и вообще вся биосфера расположены внизу, в нижней части расщелин.
А не засыпает потому что... стенки не песочные, а каменные/ледяные. Или плотность атмосферы в расщелинах нормальная, а чем выше и ближе к поверхности, тем более разряженная. А на поверхности настолько слабая, что ветра там не дуют или дуют, но не могут сдувать что-то, кроме мельчайшей пыли.
Аноним 29/01/18 Пнд 12:48:07  374463
>>374460
Нет.
Аноним 29/01/18 Пнд 12:49:51  374464
>>374457
Черная область - горизонт.
Аноним 29/01/18 Пнд 12:58:40  374467
>>374461

Предлагаю так. Планета состоит из каменного каркаса, пористого, с этими самыми щелями. И океана который на несколько сотен метров ниже каркаса. Образовалось это так: в результате магматических вулканических процессов, в толще океана образовался этот самый пористый каркас. А потом воду стало сдувать за счёт солнечного излучения и солнечного ветра, пока уровень не стал ниже уровня каркаса. При это слой биогенных материалов (углерод, фосфор и так далее) в виде ила опускался всё ниже и ниже. Такое могло получится, если планета меняла орбиту: в начале вращался вокруг одного небесного тела или была спутником газового гиганта, а потом в результате катаклизма смениал орбиту. Например вращалась вокруг Юпитера, образовался каркас, была сбита вмещательством сверттяжёлой блуждающей планеты в систему и попала на околосолнечную орбиту, с значительным эксцентриситетом. Поэтому только в глубине каркаса нормальные условия круглый год; на поверхности то слишком жарко, то слишком холодно.

Другой вариант в том, что трещины идут по экватору. Ось планеты лежит в плоскости эклептики, на полюсах то всё время холодно, то всё время жарко (годичные циклы). На поверхности очень экстремальные условия. В сети экваториальных пещер стабильные условия, вода попадает из суперштормов на поверхности. Трещины образуются от перепадов температуры. Супершторма получатся когда на одном из полюсов весной начинает таять сначала воздух, а летом застывшие в глубине водяные массивы разогреваются от приливных сил и взрывают ослабшие за лето ледяные пробки.
Аноним 29/01/18 Пнд 13:02:33  374469
>>374460
>>374460
>>374457
>>374464

ЧД это и есть горизонт. То есть само понятие "чёрная дыра" это такая область, настолько сильно притягивающая, что даже свет не вырывается. Как там внутри всё другой вопрос. То есть не важно, стоит там волшебный генератор, суператом, или равномерно распределённое вещество. ЧД это и есть сам феномен горизонта событий. Впервые ЧД нафантазировали приравниванием второй космической к скорости света. Задолго до ОТО и СТО.
Аноним 29/01/18 Пнд 13:10:05  374471
>>374442
Потому в одном случае начальная скорость 0 км/с, в другом - 7 км/с, улавливаешь?
Аноним 29/01/18 Пнд 13:10:06  374472
>>374469
Ебан? Ебан. Ладно там в сайфаче такую хуйню затирать с умным видом, но тут-то нормальные и уважаемые люди кроме катапультошизика сидят. Котлеты отдельно, мухи отдельно. Есть и чд, есть и горизонт, который эта чд "генерирует".
Вы мне так ни на один мой вопрос и не дали ответа. Аноним 29/01/18 Пнд 13:12:25  374473
Аноним 29/01/18 Пнд 13:14:43  374474
>>374471
> Потому в одном случае начальная скорость 0 км/с, в другом - 7 км/с, улавливаешь?
Нет. В начальных условиях ты ни с того, ни с сего просто попадаешь на орбиту. С нулевой скоростью. Если т ыпопадаешь на орбиту просто в обычной одежде - у тебя начальная скорость нулевая. А с какого это вдруг перепуга если ты там окажешься не в одежде в валанчике0капсуле, то с по какой-то причине начальная скорость должна оказать 7км/с? Очень странно.
Аноним 29/01/18 Пнд 13:24:37  374476
ЧД.JPG (98Кб, 564x623)
>>374472

Еблан тут ты и кореша твои, а катапультобог уважаемый человек и мать его достойная женщина.
Чёрная дыра — область пространства-времени[1], гравитационное притяжение которой настолько велико, что покинуть её не могут даже объекты, движущиеся со скоростью света, в том числе кванты самого света. Граница этой области называется горизонтом событий, а её характерный размер — гравитационным радиусом.

https://ru.wikipedia.org/wiki/Чёрная_дыра

И Ландау тоже на моей стороне, том два, параграф 102 "гравитационный коллапс сферического тела". Так что иди ты нахуй, чмыня, такс топить.
Аноним 29/01/18 Пнд 13:45:21  374481
>>374428
Репорт за что?
https://geektimes.ru/post/297621/
Последний запуск Ариана.
Аноним 29/01/18 Пнд 14:25:04  374485
1516706143257.png (336Кб, 576x590)
>>374457
Не могу нормально спать по ночам когда думаю о таком.
Аноним 29/01/18 Пнд 14:30:40  374489
>>374485
Я когда ctrl+c в SE жму то зажмуриваюсь немного как в ожидании криппи-гифки.
Аноним 29/01/18 Пнд 14:37:57  374491
>>374472
Он прав, а ты толстишь.
Аноним 29/01/18 Пнд 14:47:15  374492
>>374444
>массой до миллиарда тонн
>Увеличить ускорение от притяжения можно, когда помимо земного притяжения и само тело будет притягивать Землю.
Чё ты блять несёшь? Они и так всегда притягивают друг друга. Сам блядь всё расписал правильно, откуда вдруг эта лабуда?
Аноним 29/01/18 Пнд 15:18:11  374493
Перекрывают ли звезды нашей галактики обзор на другие звезды, которые дальше от нас? Перекрывают ли они от нас целые галактики?
Аноним 29/01/18 Пнд 15:25:55  374494
>>374493
1) Да
2) Нет, слишком маленькие угловые размеры почти у всех кроме Солнца.
Аноним 29/01/18 Пнд 15:31:52  374495
>>374494
> 1) Да
Как тогда узнают о тех звездах, что перекрыты?
Аноним 29/01/18 Пнд 15:40:45  374497
>>374495
> Как тогда узнают о тех звездах, что перекрыты?
Никак.
И уточнение про галактики: звёзды не перекрывают их геометрически своим диском, но они, как правило, достаточно яркие, чтобы телескопы не видели ничего в непосредственной близости от диска, так что какая-нибудь мелкая тусклая галактика на большом красном смещении вполне способна уйти незамеченной.
раз вы не хотите отвечать на уные вопросы вот вам простые Аноним 29/01/18 Пнд 15:46:20  374498
Я прилетел на Марс в составе экспедиции. В течении некоторого времени мы все срали в бак размером примерно один кубометр без химии и дезинфекции. К мементу старта домой, мы решили высыпать говнище в яму глубиной метра 3-5 и засыпать сверху грунтом что и сделали.
Вопрос:
Как долго выживут наши бактерии в очутившийся среде? Кто погибнит сразу а кто уживчивее?
Будут ли они адаптироваться к другим условиям?
Будут ли перекочовывать в грунт?
Будут ли мутировать?
Через 500 миллионов лет домутируют до разумных из говнобактерий?
Аноним 29/01/18 Пнд 15:51:18  374499
>>374498
>простые
Аноним 29/01/18 Пнд 16:00:25  374500
>>374495
Целый сектор нашей галактики закрыт от нас её центром, и мы ничего не знаем о том, что там.

>>374498
>Как долго выживут наши бактерии в очутившийся среде? Кто погибнит сразу а кто уживчивее?
Не знаю.

>Будут ли они адаптироваться к другим условиям? Будут ли перекочовывать в грунт? Будут ли мутировать?
Да.

>Через 500 миллионов лет домутируют до разумных из говнобактерий?
Нет.
Аноним 29/01/18 Пнд 16:10:50  374502
>>374500
понятно,лучше на чужой планете не срать чтоб не платить потом алименты.

Хорошо. Повторю первоначальные вопросы пока вы тут.
1. Прицелился я Вояджером-8 в соседнюю звезду и пульнул. Вот он летит много времени и его металлический корпус охладился. По такому случаю он стал сверхпроводником. Как изменится его полёт в магнитном поле солнца и галактики?

2. Мой спутник летает на средней земной орбите уже несколько лет. За это долгое время солнечное излучение повыбивало из его оболочки много электронов отчего он зарядился ну до фига положительно (миллионы вольт). Я решил подлететь и приконнектиться к своему спутнику на абордаж. Чо произойдёт в таком случае при приближении очень заряженного тела к нейтрально заряженному в вакууме? Что будет со связью, внешними приборами? Что будет при касании?
Аноним 29/01/18 Пнд 16:12:49  374503
3 есть ли вразумительные расчёты магнитного дирижабля и насколько это перспективно?
Аноним 29/01/18 Пнд 21:18:45  374534
>>374503
>3 есть ли вразумительные расчёты магнитного дирижабля и насколько это перспективно?
чё?
Аноним 29/01/18 Пнд 21:30:24  374535
Что дает космонавтам т.н. чепчик во время взлета-посадки? Что если без него лететь?
Аноним 29/01/18 Пнд 22:25:40  374539
>>374535
Пот не заливает глаза. Это аналог банданы или повязки.
Аноним 29/01/18 Пнд 22:29:11  374540
>>374535
А если без него полететь, мамка из ЦУПа ругать будет.
Аноним 29/01/18 Пнд 23:42:44  374544
>>374535
В него же наушники вшиты. Да и об шлем стукаться случае чего не так больно.
>>374540
Из цупа не только мамка ругать будет
Аноним 29/01/18 Пнд 23:48:20  374546
image.png (912Кб, 921x518)
>>374535
>чепчик
Аноним 30/01/18 Втр 00:03:02  374550
>>374544
>>374546
Я тут даже вспомнил, как оно называется. Это же шлемофон, ёптыть.
Аноним 30/01/18 Втр 00:06:37  374551
bullshit.webm (1312Кб, 640x360, 00:00:25)
>>374413
>некоторые объекты вроде нейтронных звёзд возможно квинтиллионы лет если не больше.

Нейтронные звезды, если на них ничего не валится в обилии, остывают очень быстро.
Аноним 30/01/18 Втр 01:24:35  374559
April 2017 Sola[...].webm (6524Кб, 640x360, 00:01:22)
>>374502
>По такому случаю он стал сверхпроводником
Нет, не стал. Сверхпроводимые свойства проявляют не все подряд элементы. В основном только в лютых сложных соединениях и при определенных условиях.

>а это долгое время солнечное излучение повыбивало из его оболочки много электронов отчего он зарядился ну до фига положительно (миллионы вольт)
Самый большой заряд на спутнике был зарегистрирован на уровне около 10 кВ.
В принципе, спутники находятся в разряженной плазме и обладают плавающим потенциалом. Приносимых ионов и электронов примерно равное количество, и только вот совсем на залитых светом аппаратах может скопиться значительных перевес электронов. При том может наблюдаться дифференциальная зарядка, так как на солнечной стороне накапливается дохуя электронов, а на теневой - нет и это довольно опасная хуйня для самого спутника.
>Чо произойдёт в таком случае при приближении очень заряженного тела к нейтрально заряженному в вакууме?
Ничо, а чо должно, если ты нейтрален?
>Что будет со связью, внешними приборами?
Какой дятел повесил наружу приборы, не оградив их от статики?
>Что будет при касании?
Ничего не будет.

Вот один дядька предлагает спутники с геостационара сводить, другим спутником. Мол специальным аппаратом стрелять по мусорным спутникам из электронно-лучевой пушки, заряжать этот мусор отрицательно, а спутник с пушкой заряжать положительно, а потом тащить на разности заряда это говно куда подальше. Довольно занятная наркоманская идея. Зато нет риска разрушить мусорный спутник, создавая еще больше обломков, воздействуя на него физически, или лазером, или еще чем. Кстати, этот метод работать будет на геостационаре, а на низких орбитах, включая твою работать не будет, так как плазма там плотнее и холоднее и аппараты, как правило, имеют нейтральный заряд.
Аноним 30/01/18 Втр 01:58:07  374563
>>374474
Само слово «орбита» подразумевает движение с такой скоростью, чтобы траектория вела не в поверхность планеты. Если ты с нулевой скоростью висишь в 400 км над Землей, то ты пусть и в космосе, но никак не на околоземной орбите.

Всегда горит, когда вижу, как журналисты путают подобные вещи и пишут про «орбиту» на суборбитальных запусках.
Аноним 30/01/18 Втр 02:26:43  374564
>>373250 (OP)
Тупой вопрос от тупсеня:
чо за хуйня с большим взрывом и всем с ним связанным? Там вроде хабл открыл интересный факт того, что в данный момент все галактики удаляются друг от друга причем чем дальше тем на более красных смещениях. И отсюда по всей видимости он и кто-то еще, с расчетами, состряпали теорию того, что когда-то вся вселенная была ну пиздец какой маленькой. Через какую литературу к этому всему прийти?
Научпоп не предлагайте, разве что такой, через который можно реально потом перейти на строгий уровень понимания (а среди научпопа, который довелось мне читать, такого было ну ооочень мало)
Cейчас читаю общий курс астрономии коновича и морозова и решаю школьные олимпиадные задачки по астрономии.
Аноним 30/01/18 Втр 03:02:54  374567
>>374564
Через ОТО, метрику Фридмана и реликтовое излучение приходишь к ΛCDM.

>Через какую литературу к этому всему прийти?
Х.З. Зельдовича наверни что ли.

Долгов, Зельдович, Сажин. Космология ранней Вселенной, например.
Аноним 30/01/18 Втр 03:46:21  374569
>>374567
cпасибо за столь меткое указание направление, это немного облегчит задачу
Аноним 30/01/18 Втр 04:06:38  374570
Почему лунную ракету назвали Сатурном?
Аноним 30/01/18 Втр 04:23:13  374571
>>374570
Потому что до Сатурнов были Юпитеры, баллистические ракеты такие.
Аноним 30/01/18 Втр 06:11:14  374576
>>374571
А у них точно есть связь между собой?
Аноним 30/01/18 Втр 06:11:28  374577
>>374559
> плазма там плотнее
> плазма
> плотнее
Мы не о жидкостях и не о металлах говорим
30/01/18 Втр 07:10:36  374584
junov-1.jpg (135Кб, 710x952)
junov-2.jpg (115Кб, 695x939)
>>374576
Самая прямая.
Аноним 30/01/18 Втр 07:14:28  374585
>>374551
>остывают очень быстро.
Эээ.. где то же писали что у них в крайне малом объёме сосредоточено очень много энергии из - за их гипер высокой плотности, при этом у них очень малая площадь поверхности в сравнении с белым карланом, у того хоть объём и намного больше, но и плотность на порядки меньше из - за чего тот энергии все таки содержит меньше.

В чем тут обсер?

Ну ок не квинтиллионы лет, но они же всё равно будут остывать дольше белых карликов?
Аноним 30/01/18 Втр 08:33:09  374603
c7a66d64436f455[...].jpg (25Кб, 600x450)
навеяно вот этими постами >>374434>>374467 и апельсином, который я съел
смотрю на апельсиновую корку и вижу, что если бы планета была поверхностью схожа с апельсином, тогда все её биомы были бы в этих ямках/конусах/карьерах/оврагах. как бы тут жизнь развивалась? я так понимаю, каждая такая анклавная долина - отдельный биом?
Аноним 30/01/18 Втр 08:34:33  374605
Если астро/космо навт встанет спиной к стенке КК и начнет стрелять из пулемета - полетит ли КК в направлении против стрельбы?
Аноним 30/01/18 Втр 08:57:37  374610
>>374605
Да. Но только если идеально направить дуло, на деле скорее всего будет отклонение на пару груадусов вперде и КК получит ещё и вращательный момент
Аноним 30/01/18 Втр 09:59:16  374624
>>374605
Ты имеешь в виду снаружи КК? Тогда да. А если внутри, то тоже да, но только потому что пули пробьют дырку в корпусе и через неё будет вытекать воздух.
Аноним 30/01/18 Втр 11:41:06  374636
от.png (38Кб, 1093x1167)
>>374605
Аноним 30/01/18 Втр 11:52:26  374639
А спутнику на светопроводах и\или со сфетопроцессором, комфортно будет в рад.поясах?
Аноним 30/01/18 Втр 17:41:43  374739
>>374577
рпределение плотности знаешь? Нет? Ну так и съеби нахуй.
Аноним 30/01/18 Втр 17:55:48  374743
const.jpg (74Кб, 810x1080)
Вопрос очень важный, но соус пока раскрыть не могу, если поможете, постараюсь раскрыть. Суть такова: нужен кто то, хорошо знающий карту созвездий. Может ли он распознать какую либо констеляцию на пикрелейтеде? Либо что то очень похожее? Может убрав звезду, заменив звезду, хз.
Аноним 30/01/18 Втр 18:01:53  374750
image.png (69Кб, 461x459)
Урка.png (143Кб, 951x577)
>>374585
>В чем тут обсер?
В нейтринном охлаждении. Урка/дурка/мурка-процессы, вот это всё. Впрочем, фотонное охлаждение у них тоже резвое.
Аноним 30/01/18 Втр 18:03:52  374753
Поясните дауну. Вот говорят, что шаттл не садился автоматически. А как это дело происходило тогда на всяких непилотируемых запусках спутников?
Аноним 30/01/18 Втр 18:04:58  374755
>>374753
А он и не садился, а сгорал в атмосфере.
А ты назови хоть один непилотируемый запуск.
Аноним 30/01/18 Втр 18:06:20  374756
15173241488110.jpg (37Кб, 810x1080)
>>374743
>но соус пока раскрыть не могу
Ну так уебывай нахуй, кому интересны твои шарады, дебила кусок.
Аноним 30/01/18 Втр 18:18:58  374766
>>374755
Ебать. Это получается они каждый раз с огурцами запускали? Не знал.
Аноним 30/01/18 Втр 18:44:54  374778
А расскажите, местные старожилы, что это за огурцач, который тут периодически упоминается, и куда его потерли, и почему?
Аноним 30/01/18 Втр 18:59:12  374785
>>374778
kerbal space programm, игрулька где огурцеподобные гуманоиды строят ракеты из готовых частей (имеющих реальные прообразы) то есть баков, движков, обитаемых и необитаемых отсеков, солнечных батарей, шлюзов, посадочных опор и прочего и развивают свою космическую программу. Искать в вг или ещё где, потёрли потому что занимала относительно много места на доске. Зря потёрли, теперь всякие дауны набегают, игра давала соображения о выводе ракет и орбитальной механике.
Аноним 30/01/18 Втр 20:36:37  374812
>>374785
Я так-то не про ksp спрашивал, уже не первый год играю. Интересовал вопрос какого хрена выпилили треды. Но то, что зря - согласен. Каким бы он ни был, это все-таки единственная в своем роде игра, нормально объясняющая орбитальную механику новичкам.
Аноним 30/01/18 Втр 22:36:33  374830
>>373939
бампуэ
Аноним 31/01/18 Срд 08:45:16  374865
Как называется орбита, когда тела имеют одинаковую орбиту, но не обязательно находятся рядом?
Аноним 31/01/18 Срд 08:49:33  374866
c070ad5b65f6528[...].jpg (86Кб, 1200x630)
>>374865
> Как называется орбита
> когда тела имеют одинаковую орбиту
...орбита?
Аноним 31/01/18 Срд 08:57:14  374867
>>374866
Ну, например, запускаем аппарат и ставим ему орбиту как у Луны, такой же пери-апогей, период, наклонение и прочее, но сам аппарат находится на произвольном расстоянии от Луны противоположный или треть витка. Так вот, как называется такая орбита относительно Луны?
Аноним 31/01/18 Срд 09:36:11  374868
>>374867
Co-orbital configuration. Тело будет называться co-orbital – как по-русски, ХЗ. Соорбитальным?
Аноним 31/01/18 Срд 09:36:45  374869
>>374867
Всё ещё не понятно. Геостационарная орбита?
Аноним 31/01/18 Срд 09:39:18  374871
>>374867
https://ru.wikipedia.org/wiki/Облака_Кордылевского
Аноним 31/01/18 Срд 09:49:45  374873
Существуют ли естественные спутники со стационарной орбитой?
Аноним 31/01/18 Срд 09:57:10  374874
>>374867
Противофазная орбита?
Аноним 31/01/18 Срд 10:45:06  374876
>>374871
очевидный фейк
Аноним 31/01/18 Срд 11:34:00  374878
>>374865
>>374867
Точки лагранжа мб?
Аноним 31/01/18 Срд 19:21:44  374926
>>374867
У них будет по меньшей мере один неодинаковый элемент орбиты.
В зависимости от выбранной системы орбитальных параметров, разными будут истиные аномалии или средние аномалии или средние долгОты (равноденственные элементы)... но если объекты в разных точках, то у них разные орбиты.
Аноним 31/01/18 Срд 20:22:29  374944
>>374873
Плутон и Харон
31/01/18 Срд 20:33:37  374946
>>374926
>если объекты в разных точках, то у них разные орбиты
Отнюдь. Орбита - это траектория.
Аноним 31/01/18 Срд 20:48:40  374948
>>374873
Земля у Луны
Аноним 31/01/18 Срд 21:25:55  374960
>>374946
Но ты не сказал ничего противоречащего.
Если объекты в одно и то же время находятся в разных точках, то у них разная траектория.
Да, чувак, траектория -- это множество точек в системе отсчёта, составной частью которой является отсчёт времени. Так что, скажем, у Grace-A и Grace-B разные траектории (что отражается в разных средних элементах в TLE), хоть и летят они аккурат друг за дружкой.
31/01/18 Срд 21:40:47  374962
>>374960
Траектория - это воображаемая линия в пространстве. Времени в определении нет.
А для орбит всё совсем просто.
Аноним 31/01/18 Срд 23:10:28  374981
>>374962
Это твоё выдуманное определение, ну или из учебника 6 класса.
Простота орбит примерно там же.
Ты можешь внятно сформулировать, что ты называешь словом "орбита"? Вот как Кеплер сформулировал, например;у него это шестерка чисел с конкретным смыслом, и разным положениями в пространстве всегда соответствуют разные шестерки чисел.
Аноним 01/02/18 Чтв 00:45:52  374998
>>374981
>Вот как Кеплер сформулировал

Это его субъективное определение, более чем модельное.
Аноним 01/02/18 Чтв 03:40:05  375051
Требуется тру ответ. Действительно высока вероятность ли ЧД, быть такой какой её описывает вращающаяся электризованная, т.е связанной с другой вселенной\ёбаным ничто\етц? Что там с временной осью?
01/02/18 Чтв 07:27:12  375061
>>374981
>это шестерка чисел, и разным положениями в пространстве всегда соответствуют разные шестерки чисел
Сам-то понял, что написал? Из этого следует, что у тела, движущегося в центральном поле тяготения при отсутствии возмущающих воздействий постоянно, непрерывно и неотвратимо меняется орбита. Что, конечно же, чушь.
Аноним 01/02/18 Чтв 07:27:51  375062
Почему ЯРД не стали применяться?
Аноним 01/02/18 Чтв 07:48:25  375065
>>375062
СССР Канаду засрал радиацией
Аноним 01/02/18 Чтв 08:38:34  375069
>>375061
Вообще-то не чушь, а так именно так и есть, поэтому элементы орбиты всегда даются на какую-то эпоху, т.е. момент времени, в который эта шестерка чисел верна.

В астрономии, например, стандарт это J2000.0, для новообнаруженных небесных тел иногда даются другие эпохи.
Аноним 01/02/18 Чтв 08:54:00  375070
>>375051
> вращающаяся
100%. Нет способа изначально получить невращающуюся ЧД, а вероятность потери углового момента в дальнейшем невелика.
> электризованная
0%. Космос в среднем нейтрален.
Аноним 01/02/18 Чтв 09:52:13  375080
А правда, что как-то на шаттле разгерметизировался сортир и космонавт был вынужден жопой закрывать его на время устранения неисправности?
Аноним 01/02/18 Чтв 10:17:12  375081
Анон, поясни за предел Канторовича
Аноним 01/02/18 Чтв 11:28:31  375084
>>375062
Опасно для экипажа и экологии
Аноним 01/02/18 Чтв 11:56:24  375089
>>375084
А ракеты на гидразине не опасно?
Аноним 01/02/18 Чтв 11:57:06  375090
>>375089
Нет. Просто не вдыхайте и не пейте его.
Аноним 01/02/18 Чтв 13:53:58  375107
>>375089
А ты считаешь, что ЯРД не дает выбросы в атмосферу при старте ракеты и наборе высоты? По-моему, ЯРД стоит использовать только в случае межпланетных перелетов, например, когда на керосине/водороде отдаляешься на определенное расстояние, а потом включаешь ЯРД, в таком случае не будет большого вреда.
Аноним 01/02/18 Чтв 14:01:10  375108
>>374948
>Земля у Луны
Нет.
Аноним 01/02/18 Чтв 14:07:03  375110
>>375081
>предел Канторовича
Нет такого понятия, алсо, Канторович к теме раздела не относится.
Аноним 01/02/18 Чтв 14:14:19  375113
>>373294
"бля, ара, чёёё с ним натури"
Аноним 01/02/18 Чтв 15:10:04  375116
>>375107
>А ты считаешь, что ЯРД не дает выбросы в атмосферу при старте ракеты и наборе высоты?
Нет. А ты считаешь, что там ТВЭЛы кусками вылетают на старте?
Аноним 01/02/18 Чтв 17:17:59  375148
>>375051
> вращающаяся электризованная

не имеет никакого отношения к

>связанной с другой вселенной\ёбаным ничто\етц?
Аноним 01/02/18 Чтв 17:36:08  375154
15169671024242.jpg (35Кб, 622x473)
При каких условиях Как-Земля может обзавестись магнитным полем с двумя осями, как у газовых гигантов и солнышка?
Аноним 01/02/18 Чтв 17:45:03  375155
>>375154
>магнитным полем с двумя осями, как у газовых гигантов и солнышка?
Аноним 01/02/18 Чтв 18:08:59  375158
>>374865
Синхронная
Аноним 01/02/18 Чтв 18:48:54  375161
sunmagneticfield.jpg (34Кб, 699x386)
>>375154
чего блядь?
Аноним 01/02/18 Чтв 18:50:35  375162
48535BD10000057[...].jpg (107Кб, 634x584)
>>375155
Аноним 01/02/18 Чтв 19:25:20  375166
>>375148
Ну, мне в ютубе сказали, что под горизонтом вращающийся электризованой есть 2 горизонт, который выкидывает вещество в белую дыру.
Аноним 01/02/18 Чтв 19:45:50  375172
image.png (17Кб, 619x337)
>>375166
1. Существование белых дыр не обнаружено даже косвенно. Даже намеков нет. Даже блядь, ну совсем-совсем ничего похожего.
2. Второй горизонт возможен у дохуя заряженных Черных дыр, однако дохуя заряженных черных дыр во Вселенной, скорее всего нет вообще. Это математическая абстракция, для разминки очень заряженных мозгов.
3. Нехуй смотреть всякое говно. Читай книги.
Аноним 01/02/18 Чтв 20:04:43  375173
1 февраля - годовщина гибели Колумбии.
Помянем.

Последний полёт космического челнока «Колумбия» начался 16 января 2003 года. Утром 1 февраля 2003 года после 16-суточного полета шаттл возвращался на Землю. НАСА потеряло связь с кораблём примерно в 14:00 GMT (09:00 EST), за 16 минут до предполагаемой посадки на ВПП №33 Космического центра имени Джона Кеннеди во Флориде — она должна была состояться в 14:16 GMT. Очевидцами были засняты горящие обломки шаттла, летящие на высоте около 63 километров при скорости 5,6 км/с. Все 7 членов экипажа погибли.
Аноним 01/02/18 Чтв 21:23:02  375186
>>375173
Чего-то тебя, мудило, не было ни 24 апреля, ни 30 июня, ни 24 октября, и 28 января ты тоже ебало не разевал. Ты сюда по ссылка из вкудахта пришел, или тебе просто сказать нехуй?
Аноним 01/02/18 Чтв 22:07:47  375191
>>375172
1.Т.е. ты утверждаешь что БД обязаны быть именно на нашей трёхмерной плоскости.
Аноним 01/02/18 Чтв 22:11:13  375192
Почему у многих боковых ускорителей тупой нос?
Аноним 01/02/18 Чтв 22:14:05  375193
>>375191
>Т.е. ты утверждаешь что БД обязаны быть именно на нашей трёхмерной плоскости.
Я утверждаю, что нехуй совать свиное рыло в калашный ряд и приписывать слова человеку, которых он не говорил.
Аноним 01/02/18 Чтв 22:15:45  375194
>>375192
У каких именно и насколько тупой?
Аноним 01/02/18 Чтв 23:00:46  375200
>>375186
> этот уязвленный вскукарек
Аноним 01/02/18 Чтв 23:21:32  375202
>>375200
Ясно, ты просто залетный говна кусок. Ехай нахуй.
Аноним 02/02/18 Птн 03:26:30  375225
>>375194
Дельта, фалкон хеви, ангара и прочее.
Аноним 02/02/18 Птн 10:12:43  375237
2.jpg (231Кб, 946x710)
4.jpg (489Кб, 900x600)
5.jpg (31Кб, 640x360)
6.jpg (924Кб, 3200x4000)
>>375225
И?
Аноним 02/02/18 Птн 10:14:51  375238
>>375191
Доказали что мы живём во вселенной с тремя пространственными и одним временным изменением.
https://nplus1.ru/news/2018/02/01/dim-gravity
Теперь петухи, пиздящие про всякую хуйню в высших измерениях, могут официально идти нахуй
Аноним 02/02/18 Птн 10:19:17  375239
>>375238
>Доказали
>nplus1.ru
ОК, лол.
Аноним 02/02/18 Птн 10:33:13  375244
>>375239
https://arxiv.org/abs/1801.08160

Вникай, маня.
Аноним 02/02/18 Птн 11:12:38  375246
Анон, немного не могу вникнут в понятия l-time i t-time у НАСА. Подскажи что к чему парой слов.
Аноним 02/02/18 Птн 11:17:09  375247
think.jpg (14Кб, 112x112)
Верно ли, что при увеличении массы черной дыры, ее средняя плотность в радиусе горизонта событий уменьшается?
Типа если посчитать среднюю плотность сверхмассивных дыр в центрах галактики, то она будет как у воды, а если для всей массы вещества во вселенной, то там вообще значения близкие к паре атомов на кубометр.
Аноним 02/02/18 Птн 11:25:48  375248
>>375247
Сам подумай, как с увеличением плотности может уменьшаться масса вещества?
Аноним 02/02/18 Птн 11:28:29  375249
>>375248
Наоборот же, больше масса, но при этом и больше радиус горизонта событий. Вопрос в том, растут ли они равномерно?
Аноним 02/02/18 Птн 11:40:15  375252
>>375247
А, глянул я тут, да, плотность уменьшается, причем на квадрат массы.
Вот тут формула https://wikimedia.org/api/rest_v1/media/math/render/svg/d12bcf24538f384eaef85e81ed5097d6b986f3aa
Аноним 02/02/18 Птн 11:43:15  375254
>>375247
> Верно ли, что при увеличении массы черной дыры, ее средняя плотность в радиусе горизонта событий уменьшается?

Да. Радиус от массы зависит линейно:

r = 2Gm/c^2

А объём сферы зависит аж от третьей степени радиуса:

V=4/3Pi*r^3

Соответственно при увеличении массы ЧД в два раза её объём увеличивается в 2^3=8 раз, а плотность в 4 раза.
Аноним 02/02/18 Птн 12:21:40  375261
>>375237
Ну туповатые же, раньше поострее все было.
Аноним 02/02/18 Птн 13:31:55  375277
Почему использование гиперголического топлива считается не очень почетным делом?
Аноним 02/02/18 Птн 13:40:35  375286
>>375261

Потому что ракета летит в плотных слоях атмосферы со скоростью пассажирского самолёта.
Аноним 02/02/18 Птн 13:42:01  375287
>>375277
Потому что вещества, которые по своим химическим свойствам подходят на роль топлива и окислителя, являются страшными ядами, а окислитель еще и норовит разъесть топливный бак. Поэтому проблемы с экологией, особенно если речь идёт о Протоне, а не топливе для спутника, и опять же не самым положительным образом влияет это дело на сложность и стоимость обслуживания и заправки.
Аноним 02/02/18 Птн 13:42:27  375288
>>375277

Couse hypergolic fuel is very toxic and have low spefic impulse.
Аноним 02/02/18 Птн 13:42:40  375289
>>375286
>>375286
Т.е. можно вообще не ебаться с аэродинамикой и пускать нагрузку в виде негабаритных кирпичей?
Аноним 02/02/18 Птн 13:43:24  375292
А ведь один бустер шаттла стоит как один флакон. А два как хэви. Бу-га-га-га.
Аноним 02/02/18 Птн 13:44:23  375293
>>375288
А какие тогда плюсы там, нет проблем с зажиганием и все?
Аноним 02/02/18 Птн 14:05:21  375305
>>375293
Ещё гиперголи не криогенные, что неплохо для космических ракет и оочень хорошо для ракет баллистических, которые должны годами дежурить заправленными. Но твердотопливные военным всё равно милее.

А ещё гиперголь при должном долбоебизме давал неплохое гуро, см. неделинскую катастрофу.
Аноним 02/02/18 Птн 15:05:19  375318
>>375289

Негабаритные кирпичи развалиться могут, так как нагрузка аэродинамическая существенная, хотя и короткая.

>>375293

Плюсы:
1. Не надо охлаждать, просто хранить.
2. Зажигается само, да.
3. Имеют высокую плотность, хранить очень удобно, ну очень.
4. Продкуты сгорания тоже имеют довольно высокую плотность, поэтому УИ в атмосфере неплохой.

>>375305
>должном долбоебизме давал неплохое гуро, см. неделинскую катастрофу.

Тут не в самозажигайках дело, если бы Неделин сидел на табуретке под заправляемым Фэлконом, то итог мог быть схожим.
Аноним 02/02/18 Птн 15:11:33  375319
>>375318
> Тут не в самозажигайках дело
В целом такое, да, может быть с любым топливом, но конкретно та авария требовала именно гипергольного топлива для развития.
Аноним 02/02/18 Птн 17:27:45  375333
>>375239
Что не так с сайтом?
Пиздежа за ними не замечал
Аноним 02/02/18 Птн 17:36:53  375335
>>373250 (OP)
Почему не поднимают ракеты до стратосферы на воздушном шаре?
Аноним 02/02/18 Птн 17:41:56  375337
>>375335
Осмиевый фонд вопросов.
Потому, что чтобы поднять ракету нужен ебических размеров шар, самый большой шар поднял килограмм сто на 40 километров, а ркеты сотни тонн весят.
А в-главных потому, что высота для орбиты составляет мизерную часть, орбита это не как высоко, а как быстро, большую часть топлива сжигают для набора боковой скорости, а не высоты, так что экономии от высокого старта будет очень мало, а гемора несоизмеримо больше.
https://what-if.xkcd.com/58/ (en)
https://chtoes.li/orbital-speed/ (ru)
Аноним 02/02/18 Птн 17:45:45  375338
>>375337
>Осмиевый
Чет я обосрался, осмий не такой уж дорогой, золото дороже.
Пусть будет плутониевый фонд.
Аноним 02/02/18 Птн 18:16:41  375347
>>375338
танталовый
Аноним 02/02/18 Птн 19:48:52  375358
>>375335

Потому что поднимает шар мало очень.
Аноним 03/02/18 Суб 00:42:40  375409
https://nplus1.ru/news/2018/02/01/dim-gravity
Анон, поясни тупому, Какое для науки и мира имеет значение новость о том, что наша вселенная четырёхмерна? Ну, то есть, кроме того, что отбросят некоторые теории гравитации кстати, раз уж отношение скорости света к скорости гравитации может отличаться от единицы на, примерно, нихуя, то что теперь будет со всякими концептами варп-двигателей и пузырей альбукерке?, что Мы ещё можем о строении вселенной узнать?
Аноним 03/02/18 Суб 01:38:03  375412
>>375409
>Какое для науки
Аккуратнее надо работать с теориями в которых подразумевается 4+n измерений. Например с теорией струн и некоторыми другими альтернативными теориями гравитации.
>Какое для мира
Никакое. Среднестатистический житель планеты будет по-прежнему сажать рис и ездить на мулах, и его внуки тоже и знать не знать про какую-то там гравитацию и сколько там измерений.
>что теперь будет со всякими концептами варп-двигателей и пузырей альбукерке?
Ничего, пространство по-прежнему гнется, в очередной раз это подтверждено.
Аноним 03/02/18 Суб 01:39:24  375413
А что с пузырем альбукерке, когда уже надувать хоть как-нибудь станут? И есть у него какое-то применение помимо сверхсветовых полетов?
Аноним 03/02/18 Суб 01:47:29  375420
>>375412
Вопрос. Там написано
> размерность нашего пространства-времени примерно равна D ≈ 4,0 ± 0,1
Что значит это 0.1? Четыре измерения возможно и ещё одно, на полшишечки?
Аноним 03/02/18 Суб 01:48:34  375421
>>375420
Ну смотри, есть еще одно, но оно не полностью ортогонально к остальным.
Аноним 03/02/18 Суб 01:55:45  375423
>>375421
Типо, не взаимодействуетслабо взаимодействует с остальными?
Аноним 03/02/18 Суб 01:58:00  375424
>>375413
>А что с пузырем альбукерке
А ничего. Не хватает точности имеющихся экспериментальных установок, что бы получить хоть какие-нибудь убедительные результаты.
>И есть у него какое-то применение помимо сверхсветовых полетов?
Ну можно будет статую Свободы переместить в Уганду, например, ради лулзов. Хотя по затрате энергии на это мероприятие, проще будет дедовским методом переместить всю Уганду в США.
Аноним 03/02/18 Суб 01:58:19  375425
>>375423
На самом деле, я в душе не ебу, просто так в разговор встрял.
Но это единственное объяснение которое приходит в голову когда слышишь дробное измерение.
Аноним 03/02/18 Суб 02:11:55  375430
>>375425
В теории струн, где 11 изменений, говорится что остальные свернулись до микроскопического уровня. Я так и не понимаю, как пространственное измерение может свернуться до микроуровня, но может это оно те самые 0.1?
Аноним 03/02/18 Суб 02:15:01  375432
image.png (215Кб, 400x300)
>>375430
По-моему пытаться представлять измерения затея безблагодатная, любая попытка оперировать более чем четырьмя - чистая математика.
Аноним 03/02/18 Суб 02:20:43  375437
>>375420
>>375430

>Что значит это 0.1? Четыре измерения возможно и ещё одно, на полшишечки?
Это значит, что мы таки живем в четырехмерном мире. 3 пространственных и одно временное. и данные показывают, что ОТО - верна.
Это 0,1 больше интересна всяким ученым, которые пытаются в разных теориях объяснить, например ускоренное расширение вселенной и\или темную материю, и работают над теориями квантовой гравитации. Например, что темная материя - есть гравитация, которая имеет взаимодействие с четвертым пространственным измерением. Эксперимент показывает, что пространственных измерений (развернутых) таки три.
При этом все это великолепие никак не влияет на теории гравитации со свернутыми измерениями. то есть это 4стопудово-исследование не запрещает Вселенной иметь сколько угодно свернутых измерений и теорию суперструн это не отменяет.
>>375432
>По-моему пытаться представлять измерения затея безблагодатная, любая попытка оперировать более чем четырьмя - чистая математика.
>По-моему Земля плоская и пытаться представлять что мы живем на шаре - затея безблагодатная.
Ну ты понял. Надо бы что бы было не "по-моему", а желательно, "наверняка", а еще лучше "стопудово - вот эвиденсес".
Аноним 03/02/18 Суб 02:22:40  375440
>>375437
Я не имел в виду, что это хуита и матан не нужен и он пиздёж, а то, что вообразить себе измерения не получится, в четырёх-то думать уже больно, а видеть так и вовсе почти нереально, а вопрос про 11 был. Только аналогии, у которых беда любых аналогий - они не точны.
Аноним 03/02/18 Суб 02:35:07  375448
>>375440
А видеть и не получится, да и не в этом дело. Важно понять, че происходит, в общих чертах и как все устроено. Если бы гравитационные волны пришли слишком поздно, или через чур ослабленные можно было бы подозревать, что часть их энергии ушло в недоступное нам измерение.
Аноним 03/02/18 Суб 10:50:59  375508
>>375172
>Голая сингулярность
Это как должно выглядеть то вообще? Некая область просранства испускающая йоба частицы с энергиями уровня \b\ на десяток порядков по больше всяких частиц типа частицы "обоже мой"?
Аноним 03/02/18 Суб 11:15:19  375510
>>375437
>Это значит, что мы таки живем в четырехмерном мире.

>Если я живу в 4-х этажке, то 11-ти этажных домов не существует!
Аноним 03/02/18 Суб 11:41:26  375514
>>375510
А я в 5-ти этажке живу.
Аноним 03/02/18 Суб 12:04:44  375515
Какую перегрузку испытывают космонавты? Слышал, что лётчики-налетчики на истребителях испытывают перегрузку в несколько раз больше, чем космонавты, так ли это? Как справляются с отрицательными перегрузками при посадке космонавта? И самый тупой: если человека перевернуть вниз головой при падении, будет ли это влиять на перегрузку или нет?
Аноним 03/02/18 Суб 12:08:35  375516
>>375337
>а ркеты сотни тонн весят
Они так весят в основном из-за топлива.
Аноним 03/02/18 Суб 12:17:38  375518
Можно сделать двигатель, работающий и на керосине и на водороде?
Аноним 03/02/18 Суб 13:14:33  375526
Почему индусы так тормозят с запуском своей навигационной системы?
03/02/18 Суб 13:27:14  375527
>>375518
Уже сделали: >>372250
Аноним 03/02/18 Суб 13:41:50  375529
>>375526
В невесомости не вытанцовывается.[/]
Аноним 03/02/18 Суб 14:31:56  375543
>>375516
При старте с 40 километров его все равно надо сотни тонн.
Аноним 03/02/18 Суб 14:34:12  375545
>>375515
3г штатные перегрузки.
При аварийной посадке по баллистической траектории до 9г.
У пилотов самолеты без проблем 9г вытворяют, могут и больше, но ограничены, чтоб пилота не вырубило.
Отрицательных перегрузок при посадке у космонавтов не бывает - они все падают жопой вниз, в том же направлении и тормозят.
Если перевернуть - будет хуево из-за притока крови к голове. При больших перегрузках даже фатальненько.
Аноним 03/02/18 Суб 16:05:05  375572
>>375515
>>375545
Добавлю лишь, что космонавты перегрузки на спину в горизонтальном положении принимают, это идеальная позиция. А у летчиков те же перегрузки в вертикальной плоскости в основном - и это уже намного тяжелее, особенно отрицательные, сердечко не справляется с подъемом крови, гроб, гроб, потеря зрения, потеря сознания.
Аноним 03/02/18 Суб 16:08:25  375573
>>375572
Потому у пилотов люто обжимающие летные костюмы. У космонавтов тоже охуенные, но не так обжимающие.
Аноним 03/02/18 Суб 16:08:48  375574
>>375526
Она нужна только военным, индусы пока не готовы экономически и политически вкладываться в оборонку.
Аноним 03/02/18 Суб 16:10:15  375575
>>375574
Чем их ГЛОНАСС не устраивает? Их Миги же все равно по нему летают.
Аноним 03/02/18 Суб 16:36:08  375584
>>375575
По нав.системе не только самолеты, но и ядерные ракеты должны летать и всякая гиперзвуковая хрень. Вдруг рашка вступит в военный союз с Китаем через 20 лет в ответ на скоординированное давление с Запада на обе страны? А на постройку спутниковой группировки нужно примерно столько времени. Не хочу в политоту скатываться, но реально спутниковая навигация это стратегическая хуйня, нужно в ней быть независимым. Также и на GPS индусам лучше не полагаться, так как США всегда могут переодеться и начать кошмарить Индию за притеснение муслимов, наг, мьянмарцев каких-нибудь, поставят свою ПРО на Шри-Ланке и будут оттуда диктовать свою волю индусам..
Аноним 03/02/18 Суб 16:37:56  375586
>>375584
>ядерные ракеты
Так это касается только ядерных держав, России и США, остальным-то нахуя?
Аноним 03/02/18 Суб 16:41:39  375589
Тупой вопрос: есть ли смысл при запуске ракеты использовать притяжение Луны?
Аноним 03/02/18 Суб 16:42:03  375590
1280px-AgniMiss[...].png (472Кб, 1280x649)
>>375586
гугли ядерный клуб, Индия имеют ядерное оружие. И еще они имеют ядерные ракеты летающие на дистанцию 10000, хотя казалось бы зачем так изъебываться, если твои потенциальные враги(Пакистан и Китай) не дальше 5000км
Аноним 03/02/18 Суб 16:44:42  375591
>>375590
Кому какое дело до страны с полутора бомбами и ржавыми ракетами? Только Россия и США обладают ядерным оружием, у остальных так, баловство. Они и не посмеют рыпнуться, потому, что когда их трухлявые ракеты будут сбиты их земля превратится в выжженный пепел.
И вообще, нахуя ядерной бомбе наведение? Ебануть куда-нибудь рядом и там пизда всему.
ГЛОНАСС и ГПС должно быть достаточно каждому. Нет, надо свою байду, галилею и прочую ебалу выводить, нам ведь так нужен синдром кесслера, пидорасы.
Аноним 03/02/18 Суб 16:48:20  375593
>>375591
синдром кесслера это страшилка для детей и политиков, как ядерная зима.
Аноним 03/02/18 Суб 16:51:40  375594
>>375593
>страшилка
Вон, уже один раз орбитально-космическое происшествие с иридиумом случилось, щас там куча говна летает.
Спутники в итоге будут либо выходить из строя, либо сильно тяжелеть потому, что надо будет обвешивать защитой от сантиметровых ошметков летящих на орбитальных скоростях.
Может и не будет космос закрыт, но у спутников жизнь подсократится и пилотируемые миссии станут гораздо более рискованными.
Аноним 03/02/18 Суб 16:55:54  375595
>>375594
То есть ты признаешь, что это страшилка. Никакой цепной реакции на орбите быть не может, люди которые говорят такую хуйню не понимают объема, в котором должна происходить эта "цепная реакция", это не чашка петри и не колба, это несколько радиусов Земли. Потом мы имеем тот факт, что от осколков можно защититься, потом временной фактор, что они будут вращаться там не вечно, а рано или поздно орбиты почти всех осколков деградируют под воздействием многих факторов, далее тот факт, что можно просто категоризировать орбиты всех осколков на орбите при помощи радаров и оптических систем и далее тот факт, что можно защититься от них каким-либо образом.
Аноним 03/02/18 Суб 16:57:46  375596
>>375594
>пилотируемые миссии станут гораздо более рискованными.
Только на орбите, а на ней давно пора перестать дрочить хуй. Вот у тебя синдром кесслера госпадя случился и как это помешает тебе периодически посылать корабли на Марс и Луну? Просто делай мощную ракету и не задерживайся на орбите Земли для ненужного дрочева.
Аноним 03/02/18 Суб 17:06:14  375598
>>375596
Так мимокрокодящим болтом прошьёт. Вероятность такого больше будет даже если без опорной орбиты улетать. Просто численно больше шансов.
Надо какие-нибудь паруса упаковывать в контейнер с магнитным замком, чтоб если спутник обесточился, у него этот замок открывался и парашют/парус выбрасывался пружиной и тот бы сводился с орбиты. Как план?
Аноним 03/02/18 Суб 17:19:34  375602
>>375598
Хуйню сказал, что у тебя там за плотность осколков в бошке, если ракета пролетающая через область размером со свою ширину на второй скорости вдруг сталкивается с осколком? Это вероятность как если пуля попадет в пулю. КРАЙНЕ МАЛА даун тупой сука.
Аноним 03/02/18 Суб 17:20:41  375603
>>375575
Индия уже переходит на закупку западных образцов.
Аноним 03/02/18 Суб 17:22:55  375604
>>375602
Сам ты тупой даун. Я знаю, что крайне мала, но когда говна полно, вероятность становится больше. И когда речь про жизни людей эту вероятность лучше не увеличивать.
Вона в МКС постоянно вхуяриваются микрометеориты хотя ею по орбите елозят, вон Иридиум ловко поймал Космос, вон спутники из строя хуй знает почему выходят (им батарейки отламывает), вон китаезы говна на орбите наплодили, все это ХУЕВО.
Вероятность того, что тебя ударит молния тоже крайне мала но ты не станешь ходить под деревьями в грозу, так и тут не надо себе говна в жопу заталкивать и плодить ненужные спутники.
Аноним 03/02/18 Суб 17:28:43  375607
123661260812082[...].jpg (733Кб, 3508x2437)
>>375604
че ты тупой такой даже для такого треда. Ты всерьез думаешь, что осколки начнут как одеяло распространяться по всем орбитам, широтам и наклонениям создавая ебучее покрывало как на картинках где осколки показаны размером с архипелаг? О господи, какие у нас гигантские спутники на этой картинке, наверное в жизни они реально так скучены, что мимо них НЕ ПРОЙТИ, ЗАКРЫВАЮТ БЛЯДЬ, ЗАГОРАЖИВАЮТ СВЕТ НАХУЙ, ЗАТМЕНИЕ ИЗ ОСКОЛКОВ СНИЖАЕТ ОСВЕЩЕННОСТЬ ПЛАНЕТЫ ГОСПОДИ! МЕГАТОННЫ ОСКОЛКОВ БРОСАЮТ ТЕНЬ НА ПЛАНЕТУ, ТОЛЬКО ВЗЛЕТАЕШЬ В КОСМОС И СРАЗУ В ТЕБЯ 1000000 пуль как из пулемета попадают за 10 секунд, масса осколков в попавших аппарат превышает массу самого аппарата аааааа
Аноним 03/02/18 Суб 17:30:09  375608
>>375604
>ты не станешь ходить под деревьями в грозу
постоянно ходил, в городе и за городом. Ни разу не ебнуло ближе чем в 5 км.
Аноним 03/02/18 Суб 17:32:24  375611
>>375607
Нет ты тупой. Через лет 20 когда из-за говнища спутники не смогут жить больше 2 лет на орбите тебе промоют мозги и ты поверишь что космос не нужен, все равно там делать нехуй.
Риск увеличится для ПКА, обязательно увеличится.
Средний срок службы КА на орбите обязательно уменьшится и будет только уменьшаться с увеличением говна на орбите, придется выводить еще больше говна и в итоге спутники просто не будут покрывать свою стоимость. И будут вещать "а все равно космос не нужен", а тебе и норм, "это не снидром кесслера, это просто косомс не нужен"
Аноним 03/02/18 Суб 17:39:08  375614
>>375611
Вот когда станут, тогда и прибежишь, сучка, а пока засунь себе эту страшилку в жопу. Это тебе промыли мозги, ты даже сам не считал вероятности, не думал что спутники сталкиваются потому что у нас орбиты выбираются с учетом факторов прохода над позициями континентов и передатчиков. И поэтому на этих хайвеях есть какие-то вероятности столкновения, но ГЛОБАЛЬНЫЙ ЭФФЕКТ невозможен. Меняешь привычку, строишь космодромы и приемо-передатчики в других местах, повышаешь орбиту за счет более мощных разгонных блоков и твой ебучий эффект кесслера оказывается пробкой на 1 дороге, которая рассосется с очередным солнечным максимумом, когда атмосфера Земли разбухнет в ответ на повышенную активность нашей звезды.
Аноним 03/02/18 Суб 17:41:31  375616
>>375614
>считал вероятности
Нахуя? События произошли и будут происходить. И если насыпать говна побольше - будут происходить чаще.
Простая.
Логика.
Попробуй, базарю, еще захочешь.
Всяко лучше, чем затыкать уши и визжать про обман.
Аноним 03/02/18 Суб 17:46:26  375618
>>375616
визжишь как сучка от страха только ты, и считать сам не хочешь. Блять сука, вот представь что я пускаю из снайперской винтовки пулю на 1 км, а у тебя есть 100000 стрелков, которые стоят на дистанции этого км и пытаются сбить пулю из своих автоматов, стреляя перпендикулярно траектории её движения. Какова вероятность, что они попадут в пулю? Вот это и есть твоя ебанутая вероятность того, что осколочек попадет в космолет покидающий Землю на 2 космической скорости.
Аноним 03/02/18 Суб 17:48:45  375620
>>375618
Все б ничего, если бы мы готовились к одному единственному полету в космос и дальше не летали бы.
Но летать-то собираемся постоянно, постоянно использовать спутники.
Теперь стреляй пейнтбольными шариками и где-то через полчаса стрельб посредине на поле будет очень много краски от сбитых шариков.
Аноним 03/02/18 Суб 17:49:51  375621
>>375618
>Какова вероятность, что они попадут в пулю?

Какова?
Аноним 03/02/18 Суб 17:53:43  375622
>>375621
Почти нулевая. А шанс попадания осколка на орбите еще ниже, так как там не 1 трек длинной в 1 км на котором все осколки сосредоточены, есть разброс по высоте, и твоя ракета может полететь вообще в специально найденное окно, в котором нет никаких осколков. А такие окна можно найти даже в самых ебанутых сценариях. Я с тем дауном даже не хочу уже говорить, он боится, что вероятность того что он произвольно дезинтегрируется может повыситься с 0.000000000000000000001% в хуй знает еще какой минусовой степени повысится на 10, то это пиздец какая страшная хуйня, которой нужно бояться.
Аноним 03/02/18 Суб 17:59:46  375623
>>375622
>Почти нулевая.
>А шанс попадания осколка на орбите еще ниже

Ты филфак заканчивал или психологический?
Аноним 03/02/18 Суб 18:09:10  375625
>>375623
А твоя мать шлюха, или просто любит сосать хуи?
Аноним 03/02/18 Суб 18:25:07  375626
>>375625

Ты мамку мою не трожь!!!!!!

Давай количественные оценки вероятности встречи с осколком на орбите, иначе всё это пустые разговоры. Понятно что КОГДА-ТО может начаться, и что орбита очищается, но в сроках вопросы. В количественных оценках. А ты пытаешься успокаивать оппонента не приводя циферок. Что шанс не 100 прц все и так знают.
Аноним 03/02/18 Суб 18:45:12  375631
Правда, что бублик гравитационно устойчив?
Аноним 03/02/18 Суб 18:46:02  375632
>>375631
Относительно долгое время, но не все время, шар устойчивее.
Аноним 03/02/18 Суб 19:12:32  375635
снимок.jpg (64Кб, 538x566)
>>375589
неа
Аноним 03/02/18 Суб 19:15:17  375636
15176743529020.jpg (66Кб, 538x566)
>>375635
обосрался чутка с разрядами, лол
Аноним 03/02/18 Суб 19:24:08  375638
>>373428
Восстановим.
Аноним 03/02/18 Суб 22:23:07  375658
Возможно ли создание очень мощного ионника?
Аноним 03/02/18 Суб 22:25:55  375659
>>375658
Смотря что в твоём понимании "очень мощный".
https://en.wikipedia.org/wiki/Dual-Stage_4-Grid
Аноним 03/02/18 Суб 22:27:40  375661
>>375659
И вот ещё.
https://en.wikipedia.org/wiki/Variable_Specific_Impulse_Magnetoplasma_Rocket
https://en.wikipedia.org/wiki/Magnetoplasmadynamic_thruster
Аноним 03/02/18 Суб 22:29:46  375662
Стикер (0Кб, 512x512)
>>375661
>https://en.wikipedia.org/wiki/Variable_Specific_Impulse_Magnetoplasma_Rocket
ВАЗИМИР

Леджит тупой вопрос: а почему ионники не сделают люто длинными как ускорители-коллайдеры, так же пижже эффективность будетю
Аноним 03/02/18 Суб 23:00:15  375665
>>375421
Но в статье речь вообще не об этом.
Это просто из разницы скорости гравитационных и электромагнитных волн вывели погрешность в определении мерности пространства.
И то хуй пойми как.

В одном месте говорят что если и есть разница между их скоростями, то не более чем на 10^-15.
А потом погрешность в мерности аж 0.1.

А вообще есть такая тема, как полушишечные измерения.
Но в опытах не было обнаружено признаков их присутствия
Аноним 03/02/18 Суб 23:51:04  375674
shop193[1].jpg (12Кб, 298x298)
>>375662
Потому что с ростом скорости выхлопа удельный импульс растет линейно, а требуемая энергия квадратично, т.е. для одной и той же электрической мощности тяга очень быстро убывает. Плюс сам ускоритель становится гораздо тяжелее и дороже.

Двигатель с охуительным удельным импульсом и так уже есть, вон он на картинке, только тяга там околонулевая и аппарат на фонариках никуда не улетит.
Аноним 03/02/18 Суб 23:53:17  375675
>>375674

Нужны батарейки из антивещества, тогда норм улетит.
Аноним 04/02/18 Вск 02:39:25  375698
>>375659
Я про тягу, хотя бы до десятки. И именно ионник, а не прочие эрд.

Кстати насчет васимра, меняемые импульс-тяга у него это так задумано или побочный эффект принципа работы?

И еще, правильно ли я понимаю, что применение эрд ограничено хуевым уровнем энергетических установок?
Аноним 04/02/18 Вск 08:08:54  375709
>>375631
Только вращающийся. Гугли метрику Керра.
Аноним 04/02/18 Вск 08:10:07  375710
125308364532423[...].jpg (77Кб, 1080x1080)
Что за ракета на этой пикче? Говорят, что Дельта 2, а мне кажется, что Атлас 5.
Аноним 04/02/18 Вск 08:32:25  375711
>>375618
>стреляя перпендикулярно
У меня складывается впечатление, что ты считаешь что ракеты улетают вертикально вверх.
Алсо, твои расчеты говно. За всю историю человечества уже случилось несколько аварий вызванных космическим мусором. Вот, например:
https://ru.wikipedia.org/wiki/%D0%A1%D1%82%D0%BE%D0%BB%D0%BA%D0%BD%D0%BE%D0%B2%D0%B5%D0%BD%D0%B8%D0%B5_%D1%81%D0%BF%D1%83%D1%82%D0%BD%D0%B8%D0%BA%D0%BE%D0%B2_%D0%9A%D0%BE%D1%81%D0%BC%D0%BE%D1%81-2251_%D0%B8_Iridium_33
Если даже считать болты равномерно распределенными по времени, этот случай уникальным, а космическую эру считать с запуска Спутник-1, то каждую секунду шанс столкнуться с космическим болтом на высоте до 1000км составляет не менее 1:2,000,000,000.
Аноним 04/02/18 Вск 08:43:51  375712
>>375710

>а мне кажется, что Атлас 5
Примерно так почувствовал? Дельта 2 это и есть.

У Атласа V вторая ступень меньше диаметром, чем первая, между ними такой конический переходник стоит.
Аноним 04/02/18 Вск 08:48:39  375713
1200px-DeltaII7[...].jpg (450Кб, 1200x1492)
>>375712
>Примерно так почувствовал?
Ага. Бустеры будто несимметрично стоят, а при гуглеже дельты обычно такая вязанка вылезает.
Аноним 04/02/18 Вск 09:37:50  375720
>>375710
Краткое руководство по отличению Атласа 5 от Дельты 2:

Цвет. Дельта синяя, Атлас оранжевый.

Боковушки. У Дельты они острые, с коническими обтекателями. У Атласа с покатыми, направленными к ракете, по типу Энергии. Ещё дельтовские сильно меньше, но по фото это не определишь.

Вторая ступень Атласа меньше диаметром, как уже сказали.
Аноним 04/02/18 Вск 09:42:10  375724
AtlasV541underw[...].jpg (641Кб, 2000x3000)
>>375720
Цвет же покрасить можно, что за аргумент?
Боковушки да, отличные.
Но вторая ступень вот у атласа такая же.

Было б круто увидеть их вместе на одном чертеже в одних габаритах чтоб прочувствовать так сказать.
Вроде были такие пикчи где ракеты в одном масштабе.
Аноним 04/02/18 Вск 10:13:57  375730
>>375724
>Цвет же покрасить можно, что за аргумент?
Иди, крась, умник.
Аноним 04/02/18 Вск 10:15:09  375732
>>375730
Че я то сразу? Их красят, хотят - в синюю, хотят - в оранжевую с серым. Это ж не цвет люминя или чего там
Аноним 04/02/18 Вск 10:26:38  375734
AtlasEELVfamily[...].png (42Кб, 480x480)
>>375724
Второй ступени тут как раз не видно, она такая же узкая, но под удлиненным обтекателем.
Аноним 04/02/18 Вск 11:58:30  375753
AtlasV(401)onit[...].jpg (1276Кб, 3000x2000)
>>375724
Да, забыл об этом.

В отличие от многих ракет Атлас 5 имеет два варианта головного обтекателя: диаметром четыре метра и диаметром пять метров. Четырёхметровый обтекатель (см. пикрил) оставляет вторую ступень открытой, а пятиметровый (как у тебя — это Атлас V 541, запустивший MSL/Curiosity) её закрывает.
Аноним 04/02/18 Вск 12:40:01  375763
>>374402
>в капсуле уже можно, даже проще, чем с орбиты, потому что скорости гораздо ниже.
Не факт. При сходе с орбиты мы входим в атмосферу почти горизонтально, из-за чего можем тормозить плавнее, особенно если спускаемый аппарат имеет подъёмную силу и активное управление как когда-то КM Аполлонов и сейчас СА Союзов.

А вот если падать вертикально вниз с большой высоты, то не успеешь нормально затормозить до попадания в сверхплотные слои. В итоге пиковая перегрузка и пиковая тепловая нагрузка может выйти совершенно конской. Чтобы понять точнее нужно посчитать, но этот эффект совершенно реален: у Шепарда (апогей 185 км) максимальная перегрузка была 11.6 g, а при даже неуправляемом спуске СА Союза выходит девять.
Аноним 04/02/18 Вск 13:00:14  375768
В википедии написано что давление в лунном модуле было 33кПа. Это примерно как на Эвересте. Не маловато?
Аноним 04/02/18 Вск 13:13:39  375771
>>375768

Маловато для чего?
Аноним 04/02/18 Вск 13:19:33  375773
>>375771
Для космонавтов.
Аноним 04/02/18 Вск 13:35:46  375775
>>375773

Там же чистый кислород был. Нормально.
Аноним 04/02/18 Вск 20:16:21  375881
705.png (229Кб, 630x345)
Если скорость света в вакууме взяли за максимально возможную т.к. единица света не имеет веса, но вроде как потом оказалось что вес она имеет, и даже открыли что есть цастицы которые быстрее света летят, получается что основной принцип ОТО нарушен? И Е != МЦ^2
Аноним 04/02/18 Вск 20:22:59  375884
>>375881
Да, получается.
Но, к счастью, фотон не имеет массы покоя и не обнаружили частиц быстрее него, так что в этой вселенной всё в порядке.
Аноним 04/02/18 Вск 20:23:54  375885
>>375884
Я не поняо, поясни пожалуйста.
Аноним 04/02/18 Вск 20:28:26  375887
>>375885
Если фотон имеет массу покоя и есть частицы быстрее него, значит фундаментальная физика обосралась и много других физических законов надо пересматривать.
Но фотон не имеет массы покоя и частиц быстрее него не обнаружено. Поэтому с законами все в порядке и пересматривать их не надо.
Аноним 04/02/18 Вск 20:30:04  375889
>>375887
Я слышол что нейтрино быстрее (немношко).
Т.е. мистер свет не может остановится, а может только идти? А если его скомпенсировать обратной силой в противоположном направлении?
Аноним 04/02/18 Вск 20:33:47  375892
>>375889
>Я слышол что нейтрино быстрее (немношко).
Нет, не быстрее.
>Т.е. мистер свет не может остановится, а может только идти?
Верно
>А если его скомпенсировать обратной силой в противоположном направлении?
Приложить к фотону силу ты можешь только частицей или гравитацией. В сильном гравитационном поле скорость света может исказиться сильно. При столкновении фотон поглотится и возможно переизлучится.
Аноним 04/02/18 Вск 20:34:58  375893
>>375889
>нейтрино быстрее (немношко)
Нет, не быстрее.

В одном-единственном эксперименте измерительное оборудование было хуево подключено и выдавало неправильные данные, которые могли быть интерпретированы как существование сверхсветовых нейтрино. В реальности ничего такого не было, и после правильного подключения все заработало как надо, но журналистов уже было не остановить.
Аноним 04/02/18 Вск 20:44:43  375899
>>375893
>>375892
Cпасибо, а то я на статьюю о том что открыли частицу быстрее света отцкрыли.
Аноним 04/02/18 Вск 21:05:45  375908
ignition.png (92Кб, 632x266)
Годная ли книжка? Хочу изучить больше про движки и историю топлива для ракет.
Аноним 04/02/18 Вск 21:12:19  375910
15171235384270.png (9884Кб, 3200x1800)
Рекветирую фотки планет в ЕСТЕСТВЕННЫХ ЦВЕТАХ, без усиления подсветки цветокоррекции. Например, как выглядит Ганимед ирл? А Луна? Говорят, она ИРЛ, серо-коричнево-розовая. А каков Плутон?
Аноним 04/02/18 Вск 21:16:38  375912
>>375910
На Луну ты можешь сам посмотреть. А так ищи raw фотографии
Аноним 04/02/18 Вск 21:32:09  375925
!!!!!.png (1375Кб, 1360x643)
>>375912
Аноним 04/02/18 Вск 21:36:18  375928
fukken lold.png (230Кб, 640x400)
image.png (317Кб, 824x1356)
>>375925
>Яндекс
>Яндекс картинки
Ты бы еще в поиске мейлру на русском искал.
Хотя бы так надо (и не в картинках поиск)
Аноним 04/02/18 Вск 21:37:15  375929
Apollo15Souther[...].jpg (301Кб, 1188x1140)
JohnW.Youngonth[...].jpg (1693Кб, 3928x3928)
Apollo15Craters[...].jpg (495Кб, 1584x1586)
Apollo15ALSEP&L[...].jpg (129Кб, 904x913)
>>375912
Луна выглядит серой, но на самом деле она серо-коричневая, хотя не везде и не под всяким углом зрения, у неё свои загоны с отражением света. Можно смотреть фотки Аполлонов, они все довольно реальные.
Аноним 04/02/18 Вск 21:39:01  375931
>>375929
Я слышал что на луну не летали и фото-фейк с парамаунт-пикчерз.
Аноним 04/02/18 Вск 21:40:40  375932
>>375931
Хуево слышал. Лапшу тебе на уши навешали, а ты и поверил.
Студия Метро-Голдвин-Майер же.
Аноним 04/02/18 Вск 21:54:20  375946
!!!!!.png (1522Кб, 1360x643)
15177592183090.png (66Кб, 847x521)
>>375928
Да вы, блядь, издеваетесь
Аноним 04/02/18 Вск 21:56:00  375947
>>375946
Не в картинках ищи, а веб поиск.
Аноним 04/02/18 Вск 22:00:41  375953
>>375947
Покажи мне два скрина - куда мне нужно нажать и что у тебя получилось после нажатия
Аноним 04/02/18 Вск 22:01:18  375954
>>375953
На твоём дже скрине нажми "всё."
Аноним 04/02/18 Вск 22:05:36  375956
image.png (329Кб, 1051x1356)
>>375953
>>375946
Ты не очень сообразительный, да?
Аноним 05/02/18 Пнд 01:53:56  376031
>>375908
Да, сверхгодная. Есть в интернете, искать Ignition.epub.
Аноним 05/02/18 Пнд 02:07:03  376033
9VFfPg8rQpk.jpg (45Кб, 506x508)
Можно ли узнать, что на планете было применено нейтронное оружие? Например, прилетаем на планету человеческую-колонию или просто чужой мир, а он мёртвый - флоры и фауны нет, а техника и прочее - есть в целости и сохранности.
Ну так что, можно ли узнать, что это была именно нейтронная бомба? А эпицентр взрыва (или эпицентры)? А мощность взрыва? А давность события?
Аноним 05/02/18 Пнд 02:16:08  376034
>>376033
Хинт: нейтронные бонбы нихуя не сделают. Поубивают кучу высших организмов. Точечно. Океаны ты не стерилизуешь.
Чтоб убить все живое надо хорошо радиацией от джета ближнего пульсара пройтись.
Аноним 05/02/18 Пнд 03:12:17  376041
>>376034
>Чтоб убить все живое
А подземное дерьмо в пещерах на глубине н км, питающееся от геотермальной и т.п.?
Аноним 05/02/18 Пнд 03:13:32  376042
>>376034
>ближнего
Интересно насколько все живучее этот пульсар должен быть близко?
Аноним 05/02/18 Пнд 03:17:13  376043
YzSMES255Y.jpg (168Кб, 714x960)
>>376034
Ладно. Допустим, была живая планета с цивилизацией уровня землянского средневековья. Замки, статуи, мосты, пристани, дома - это всё построено. Потом нейтронная струя убивает всё живое в воздухе, в реках, озёрах, морях, на суше и немного в почве, но окена только до олределённой глбины - какой глубины?
Что будет с трупами всех животных и растений? Останутся лежать вечно неразлагаемыми - бактерии-то тоже сдохли?
И повторяю часть вопросов из оригинального поста
> Ну так что, можно ли узнать, что это была именно нейтронная бомба? А эпицентр взрыва (или эпицентры)? А мощность взрыва? А давность события?
Аноним 05/02/18 Пнд 04:09:37  376049
мочераторов поменяли или все теже тупые пидары что и два года назад?
Аноним 05/02/18 Пнд 05:28:42  376060
>>376043
Бактерии, мне кажется, вообще хуй когда сдохнут
Аноним 05/02/18 Пнд 07:14:23  376067
>>376060
Натяни планету приливными силами белого карлана\нейтронки\чд. Подставь планету под красный сверхгигант, ударь планету другой планетой и т.п. Думаю сдохнут даже те что в недрах земной коры на н км.. наверное сдохнут. Ну чита на неуязвимость у них же не прописано? Но конечно да, живучести бактерий любая сложная структура позавидует, как органическая так и техногенная.
Аноним 05/02/18 Пнд 10:30:15  376085
>>375929
Какой глубины кратеры на №3? Так, примерно, можете почувствовать?
Аноним 05/02/18 Пнд 10:31:01  376086
>>376085
Примерно чувствую что от 2см до 2км.
Аноним 05/02/18 Пнд 10:43:58  376087
>>376085
Так примерно чувствую, что 3,3 км и 3,6 км. Навскидку так.
Аноним 05/02/18 Пнд 11:00:16  376088
>>376043
>Что будет с трупами всех животных и растений? Останутся лежать вечно неразлагаемыми - бактерии-то тоже сдохли?
Некоторые бактерии точно останутся. Расплодятся и съедят твои трупешники.

Узнать скорее всего можно, каким-нибудь изотопным анализом. Так историю солнечной активности узнают, например. Эпицентры взрывов и всё такое тоже можно будет прикинуть.
Аноним 05/02/18 Пнд 11:10:09  376089
Для пакетной схемы масса второй ступени считается как масса центральной ступени - масса топлива, расходованного до отделения первой ступени? Я правильно понял?
Аноним 05/02/18 Пнд 11:41:39  376097
15174134792560.jpg (213Кб, 1280x853)
Как обстоят дела с жидкой водой на других планетах?
Берём некую Как-Землю. Жизнь полностью совместима с земной - углеродная, можно есть. Флора и фауна в наличии, но... вспомнил я опыт из школы. Берём пустую пластиковую бутылку, кидаем в неё спичку и наливаем воду. Спичка вплывёт. Теперь берём и плотно закручиваем крышку на бутылке до герметичности. Спичка всё ещё не тонет. Теперь сжимаем бутылку - нетонущая деревянная спичка начинает тонуть. Чому так? Я не помню объяснения.
А если на других планетах при нормальных условиях вода такая же неудобная для человека? Что же тогда. там и лодки и пловцы будут тонуть?
Аноним 05/02/18 Пнд 11:59:14  376115
>>376097

Есть такое. Дело в том, что в болотах особая, разуплотнённая вода. Такая вода лёгкая, и в ней все захлёбываются. Потому болот так боятся. Если сильные перепады давления и низкое притяжение, то вода неизбежно разуплотняется.

>>376089

Что именно ты считать собрался, для какой формулы?
Аноним 05/02/18 Пнд 12:40:27  376120
>>376115
Для формулы Циолковского. Я хочу узнать m0 второй ступени.
Аноним 05/02/18 Пнд 12:46:24  376123
У меня вопрос идущий в разрез с линией партии желаниями илона святого маска.
Как я понимаю, он предлагает своими технологиями уменьшить цену запусков, то есть увеличить количество запусков за те же деньги.
А развене будет ли прямым следствием этого (при нынешней надежности в целом всей космической программы), тот факт, что космический мусор станет копиться на орбите быстрее?
Спасибо.
Аноним 05/02/18 Пнд 13:08:19  376133
>>376085
Кармайкл - 3800 метров, Хилл - 3300 метров.
Аноним 05/02/18 Пнд 13:24:51  376142
>>376120
>>376115
Точнее, меня интересует полутороступенчатый Атлас. Мне стало любопытно, какую идеальную конечную скорость наберет такая ракета.
Там стартовая масса около 118 тонн, а в качестве первой ступени сбрасываются только стартовые ускорители массой 7 тонн.
Нужна масса второй ступени. Просто отнять от полной массы 118 тонн 7 тонн стартовых двигателей - выйдет как-то кисло. Уверен, что расходованное к моменту разделения топливо центрального блока должно учитываться как первая ступень вместе со стартовыми ускорителями.
Аноним 05/02/18 Пнд 13:52:12  376158
>>376142
Масса второй ступени = масса ракеты - ( масса стартовых двигателей + топливо, израсходованное до разделение).

Боль в том, чтобы рассчитать характеристическую скорость первой ступени до разделения, ибо там движки с разной тягой и разным удельным импульсом. Формула Циолковского, по крайней мере в базовом виде, под это не заточена.
Аноним 05/02/18 Пнд 13:58:10  376164
>>376142

Тут нужно считать следующими этапами:
1. До сброса ускорителей, м0 это масса ускорителей плюс масса центрального блока с остатками топлива, а скорость истечения-тут всё ещё сложнее, так как скорости истечения у водородного мотора и у бустера разные и от высоты зависят, и зависят тоже по-разному.
2. После всё как обычно.

Вообще выход из атмосферы и тангаж самая сложная часть расчёта, и она больше всего играет роль.
Аноним 05/02/18 Пнд 13:59:29  376168
>>376164

И первый этап считать лучше итерациями. Тупо считать характеристическую скорость по ф-ле Циолковского это для второй ступени только прокатит.
Аноним 05/02/18 Пнд 13:59:58  376169
>>376123
Будет. Не за что.
Аноним 05/02/18 Пнд 14:07:18  376170
>>376123

Он предлагает, например, взять БФС и устроить субботник на орбите.
Аноним 05/02/18 Пнд 14:34:08  376179
Анон, есть очень известный игровой симулятор (который отсюда выпилили), там при постройке ракеты приходится рассчитывать характеристическую скорость ракеты, чтобы удостовериться, что она сможет доставить что-нибудь до орбиты. Расчет этой самой характеристической скорости делаем самый примитивный через встроенный аддон - формула Циолковского для всех ступеней (начиная с верхней), значения УИ для всех двигателей берется при полёте в вакууме.

Отсюда берется интересное наблюдение - со всеми необходимым аддонами на реализм (размеры планеты, аэродинамика, поведение двигателей) получается, что при запуске даже с широт Плесецка, если мы летим на восток, ракете достаточно обладать 9км/с этой самой характеристической скорости, чтобы доставить груз на орбиту при ведении её ручками (т.е. криво и косо не по самой оптимальной траектории). Конечно, это работает не всегда, только если у нас сбалансированная ракета вроде Союза, Протона или F9 - без водородных движков на первой ступени, у которых УИ может скакать от 350 до 450 секунд, или верхней ступень с микроскопической тяговооруженностью, но если эти условия соблюдаются, то 9 км/с для выхода на орбиту это как правило большого пальца.

Вопрос - насколько этот простой подсчет по формуле Циолковского, работающий в игрульке, применим к оценке производительности реальных ракет, если знать про ограничения в применении? Я про те же водородники, которые имеют бОльшие гравитационные потери и простое ограничение по массе ПН, если вторая ступень очень слабенькая по тяге и просто не сможет успеть округлить орбиту до падения на Землю.
Аноним 05/02/18 Пнд 14:34:52  376180
О2.jpg (17Кб, 550x212)
>>376158
>>376164
Спасибо.
>>375768
недомогания в горах связаны прежде всего с недостатком кислорода, что влияет на дыхательную и нервную системы. [1]
В лунном модуле газовая среда состоит только из кислорода. На Эвересте азот и кислород. Кислорода в нормальной земной атмосфере по объему 21%. Так как
Парциальное давление любого газа в газовой смеси представляет собой часть общего давления газовой смеси, приходящаяся на долю этого газа в соответствии с его процентным содержанием. [2]
То получаем, что парциальное давление кислорода на Эвересте 21%33кПа = ~7 кПа или ~52 мм. рт. ст. Проверяем табличкой: на 8км парциальное кислорода 56 мм. рт. ст или 7,4 кПА.
На уровне море парциальное кислорода 159 мм рт. ст.(21%
760 мм. рт.ст.) или 21 кПа. А в лунном модуле парциальное давление кислорода(а другого газа, создающего давления нет) 33 кПа. Кислородное голодание астронавтом точно не грозило.

1 http://alp.org.ua/?p=36901
[2] https://studfiles.net/preview/2137801/page:5/
Аноним 05/02/18 Пнд 14:55:28  376189
>>376115
>Дело в том, что в болотах особая, разуплотнённая вода. Такая вода лёгкая, и в ней все захлёбываются. Потому болот так боятся.
Естествознание уровня /spc/
Аноним 05/02/18 Пнд 15:48:15  376206
Получается, если в Орлане заискрит проводка - космонавт сгорит как спичка в несколько секунд как на Аполлоне? Правильно понимаю? Там же тоже чистая кислородная среда при 40кПа, это даже больше чем у Аполлонов
Аноним 05/02/18 Пнд 18:45:33  376281
>>376206
>Получается, если в Орлане заискрит проводка - космонавт сгорит как спичка в несколько секунд как на Аполлоне?

Для этого надо чтоб там что-то хорошо горючее было. А что там такого может быть я даже не знаю. Крестик, волосы космонавта, иконка, лампадное масло, например.

>как на Аполлоне?

Там было атмосферное давление и чистый кислород.
Однако во время тренировок на земле и при подготовке к старту использовать пониженное давление внутри кабины было нельзя, так как корабль был рассчитан на избыточное давление изнутри, а не снаружи. Во время тренировки 27 января давление кислорода внутри корабля было даже выше атмосферного.
Аноним 05/02/18 Пнд 18:55:15  376291
06030201[1].jpg (169Кб, 600x600)
Если есть главная последовательность, то почему нет побочных? Ну или, а с хуёв ли она главная? Может быть есть другие, просто не открытые нами, а та, что есть у нас - крайне редкая, всратая и вообще особенная?
Аноним 05/02/18 Пнд 19:08:29  376297
>>376291
>Ну или, а с хуёв ли она главная?

С того, что описывает светимость обычной звезды получившейся из межзвёздного газа.

>Может быть есть другие,

Почитай учебники по астрофизике и подумай ещё раз.
Аноним 05/02/18 Пнд 19:09:43  376298
image.png (782Кб, 1044x1152)
>>376206
У Аполлона 1 экипаж сгорел больше от распиздяйства, чем от случайной искры, мозгами нужно было думать, при выборе материалов, а не жопой.

>Заискрит проводка.
Маловероятное событие, больше волноваться стоит дырку в скафандре. В Орланах нагуляли уже очень сильно за сотню.

>>376291
>Если есть главная последовательность, то почему нет побочных?
Вообще-то есть.
Главная последовательность - это только вот эта сплошная кривая, все, что вне ее, как ты выразился, "побочные".

>а с хуёв ли она главная?
потому что на ней происходит эволюция звезд с термоядерным синтезом, причем этот жизненный путь проходят все звезды, а дальше уже могут быть варианты. Поэтому она главная.
>Может быть есть другие, просто не открытые нами, а та, что есть у нас - крайне редкая, всратая и вообще особенная?
Законы физики везде одинаковые и все звезды, что мы наблюдаем ведут себя строго говоря одинаково, для своего типа.
Аноним 06/02/18 Втр 10:50:17  376542
15175433671820.gif (350Кб, 263x239)
есть красные гиганты и красные карланы.
есть синие гиганты.
бывают ли синие карланы?
Аноним 06/02/18 Втр 12:53:06  376588
>>376542
>бывают ли синие карланы
Нет, не бывает.
Аноним 06/02/18 Втр 13:39:03  376618
>>376067
Да нихуя им не будет.
Споры бактерий даже в дипспейсе десятилетиями живут.
Даже видел расчет, согласно которому на Вояджере до сих пор должны оставаться несколько сотен тысяч бактерий.

А современные амс просто так стерилизуют?

Если планету ударить планетой, то миллионы камней превратятся в осеменительные бомбы, которые при попадании в благоприятную среду начнут размножаться.
Аноним 06/02/18 Втр 13:56:01  376624
>>376618
>дипспейсе
Значение знаешь?
>Споры бактерий даже в дипспейсе десятилетиями живут.
Пруфы на это.
>Даже видел расчет, согласно которому на Вояджере до сих пор должны оставаться несколько сотен тысяч бактерий.
В студию.


Аноним 06/02/18 Втр 16:29:38  376709
Что будет, если попасть под струю ионника?
Аноним 06/02/18 Втр 16:34:05  376713
>>376709
В тебя попадут ионы.
Аноним 06/02/18 Втр 16:46:18  376719
>>373250 (OP)
Смотрю это https://www.youtube.com/watch?v=FOMNTtgOOSE и возник вопрос.

Теория большого взрыва теперь официальная и доказана однозначно? Как так?

Второй вопрос. Недавно открыли гравитационные волны. Где гарантия, что это именно то, что нужно, а не слив унитаза?
Аноним 06/02/18 Втр 16:47:30  376720
>>376713
Каковы последствия?
Аноним 06/02/18 Втр 16:57:41  376723
>>376720
Ожог, глубокий, сквозной.
Аноним 06/02/18 Втр 17:03:24  376733
>>376723
а в него ебать можно будет?
Аноним 06/02/18 Втр 17:17:51  376744
>>376723
Схуяли? Почти сразу после решетки заряда уже нету, пробивать неоткуда.
Аноним 06/02/18 Втр 17:21:38  376745
>>376723
https://www.reddit.com/r/askscience/comments/5491xz/is_the_exhaust_from_an_ion_thruster_hot_reactive/
Пиздеть не надо тут.
Аноним 06/02/18 Втр 17:26:06  376747
>>376733
>>376744
>>376745

Ну хоть мой ответ помог найти ответ, уже неплохо.
Аноним 06/02/18 Втр 17:26:34  376748
>>376747
на поставленный вопрос
Аноним 06/02/18 Втр 18:01:15  376782
Поясните за движение на околосветовых скоростях. По идее же, даже если не учитывать перегрузки экипажа, любая пылинка распидорасит корабль, разве нет? Так какой тогда смысл двигаться вообще в этом направлении?
Аноним 06/02/18 Втр 18:07:46  376785
>>376782
ну как бы хочется лететь не по тысяче лет до ближайших звезд
если уж сможем разогнаться до околосвета, то с пылью и перегрузками как-нибудь разберемся. Можно магнитное поле создавать какое-нибудь для защиты от пыли, людей в гель помещать, кровь заменять раствором от перегрузок, я не ебу какие технологии там появятся в будущем
Аноним 06/02/18 Втр 18:51:12  376807
>>376719
>Теория большого взрыва теперь официальная
Что значит официальная? Есть неофициальные теории?
>и доказана однозначно?
Ой, вей. Есть теория тяготения Ньютона, есть теория тяготения Эйнштейна. Их как считать? Доказанными, или не очень? Они обе рабочие (в том смысле, что обладают предсказательной силой) и применимы в определенных условиях. Как так?


>Где гарантия, что это именно то, что нужно, а не слив унитаза?

Ребята, у нас теория гравитации, которую мы проверили стописят раз, по этой теории должны существовать гравитационные волны. Я тут накидал на бумажке всякую магию, недоступную быдлу расчеты, мы можем построить такой-то YOBA прибор и зарегистрировать эти волны. Правда все это на грани чувствительности приборов, но может не сейчас, а через много лет, мы надежно зарегистрируем гравитационные волны.
Проходит 25 лет упорной работы, настройки и усовершенствования прибора, вложены 4 сотни миллионов долларов, в течении этого времени происходят регистрации сливов воды в унитазах, проходящих поездов, проезжающих машин, землетрясений, кряканья уток, все это нахуй выбрасывается на помойку, как не релевантные события, которые ну никак не совпадают с теоретическими ожиданиями даже близко. Пока наконец-то не получен надежный результат, который не оглашается полгода, который тратят на сопоставление фактов и выяснения природы и примерное местонахождение источника. Лишь бы только не обосраться, как ребята из BICEP2. И вот наконец пресс-релиз. Новые регистрации, нобелевская премия и под занавес, слияние двух нейтронных звезд, для которых нашли местоположение и наблюдали всем миром всеми возможными приборыми, почти на всех диапазонах электромагнитного спектра.

>Где гарантия, что это не слив унитаза?
Тогда надо было строить детектор слива унитаза.
Аноним 06/02/18 Втр 19:18:35  376830
Анон, есть очень известный игровой симулятор (который отсюда выпилили), там при постройке ракеты приходится рассчитывать характеристическую скорость ракеты, чтобы удостовериться, что она сможет доставить что-нибудь до орбиты. Расчет этой самой характеристической скорости делаем самый примитивный через встроенный аддон - формула Циолковского для всех ступеней (начиная с верхней), значения УИ для всех двигателей берется при полёте в вакууме.

Отсюда берется интересное наблюдение - со всеми необходимым аддонами на реализм (размеры планеты, аэродинамика, поведение двигателей) получается, что при запуске даже с широт Плесецка, если мы летим на восток, ракете достаточно обладать 9км/с этой самой характеристической скорости, чтобы доставить груз на орбиту при ведении её ручками (т.е. криво и косо не по самой оптимальной траектории). Конечно, это работает не всегда, только если у нас сбалансированная ракета вроде Союза, Протона или F9 - без водородных движков на первой ступени, у которых УИ может скакать от 350 до 450 секунд, или верхней ступень с микроскопической тяговооруженностью, но если эти условия соблюдаются, то 9 км/с для выхода на орбиту это как правило большого пальца.

Вопрос вот в чем - насколько этот простой подсчет по формуле Циолковского, работающий в игрульке, применим к оценке производительности реальных ракет, если знать про ограничения в применении? Я про те же водородники, которые имеют бОльшие гравитационные потери и простое ограничение по массе ПН, если вторая ступень очень слабенькая по тяге и просто не сможет успеть округлить орбиту до падения на Землю.
Аноним 06/02/18 Втр 19:29:05  376835
>>376782
Перегрузки зависят от ускорения, а не от скорости.
Пылинка распидорасит, да
Аноним 06/02/18 Втр 20:24:01  376910
>>376835
Пылинка в излучение превратится, нет?
В этих тредах писали что на околсветовых скоростях корабль будет иметь дело не с потоком песчинок, вращающихся в него, а с потоком жёсткого излучения
Аноним 06/02/18 Втр 20:30:42  376928
>>376910
С хуев блять макроскопический кусок пыли в излучение превратится?
Аноним 06/02/18 Втр 20:58:20  376997
>>376928
Из за высокой энергии столкновения?
Аноним 06/02/18 Втр 21:15:22  377054
Как спутники держатся на орбите и не падают, что их держит? Так и не понял. школота 15 лет не обоссыте
Аноним 06/02/18 Втр 21:47:17  377156
>>377054
Не ну это даже для треда тупых вопросов слишком
Аноним 06/02/18 Втр 21:53:08  377184
>>377054

Они падают. Они по кругу падают. Их центобежная сила держит.
Аноним 06/02/18 Втр 22:34:06  377330
>>377054
Они вообще в атмосфере летают, как и МКС, вполне официально. Весть "космос" большое наебалово.
Аноним 06/02/18 Втр 23:00:07  377473
>>377054
Ничего их не держит, скорость просто слишком большая, траектория падения не пересекает плотные слои атмосферы, поэтому и наворачивают круги. Чуть чуть больше скорость и вообще могут улететь с орбиты Земли.
Аноним 07/02/18 Срд 00:26:18  377989
>>376782
http://antihydrogen.livejournal.com/46578.html
Аноним 07/02/18 Срд 14:46:00  379090
Поясните ньюфагу что надо вкурить чтоб быть свои в спейсаче в треде с ракетами?
Аноним 07/02/18 Срд 14:55:29  379106
>>379090
Поиграть в огурцов.
Аноним 07/02/18 Срд 14:56:13  379108
>>379106
Наиграл 50 часов. Что дальше?
Аноним 07/02/18 Срд 15:15:23  379136
>>379108
Наиграть 500 часов.
Аноним 07/02/18 Срд 15:20:04  379142
>>379136
А если серьезно?
Аноним 07/02/18 Срд 17:48:53  379273
>>379090

К огурцам хорошо бы навернуть орбитальную механику, и историю мирового ракетостроения.
Аноним 07/02/18 Срд 18:11:40  379280
>>373250 (OP)

Что это такое? >>378638 (OP)
Аноним 07/02/18 Срд 19:36:24  379334
Там на педивикии есть информация о рн сатурн-5.
SA-514 и SA-515 написано есть, но не испольовались. Они их типа хранили какое-то время ? Или до сих пор хранят ? Сколько у них вообще "срок годности ?
Аноним 07/02/18 Срд 21:03:21  379411
>>379334
До сих пор хранят. В качестве экспонатов.
>Сколько у них вообще "срок годности ?
Да ни сколько. При соответствующих условиях и регламентах хранения могут быть использованы в течении месяцев и даже лет. В условиях музея - только если разобрать и собрать заново, попутно оценивая состояние каждого элемента комиссией.
Аноним 07/02/18 Срд 21:03:58  379413
>>379334
>Они их типа хранили какое-то время ?

Вроде до сих пор хранят.

>Сколько у них вообще "срок годности ?

Вполне возможно, что их можно запустить, если пусковая сохранилась, после минимальной проверки и обслуживания каких-нибудь резиновых уплотнителей и пластиковых деталюх, которых там мало очень. Но там могли быть, скажем, алюминий-скандиевые сплавы, а они теряют прочность даже если лежали на складе. Хотя в то время вроде не было таких сплавов, они в 80-е появились, а в 90-е стала известно о их старении без нагрузок.
Аноним 07/02/18 Срд 21:07:58  379418
>>379413
>если пусковая сохранилась
Пусковую переколхозили для запуска Пейс-шатлов, как и краулеры для вывоза РН на стол.
Аноним 07/02/18 Срд 21:11:25  379423
>>379334
>SA-515
Третью ступень этой дилды переколхозили в Скайлэб.
ПЕРЕКАТ Аноним 08/02/18 Чтв 02:16:25  379585
ПЕРЕКАТ >>379584 (OP)
https://2ch.hk/spc/res/379584.html
ПЕРЕКАТ >>379584 (OP)
https://2ch.hk/spc/res/379584.html
ПЕРЕКАТ >>379584 (OP)
https://2ch.hk/spc/res/379584.html
ПЕРЕКАТ >>379584 (OP)
https://2ch.hk/spc/res/379584.html
ПЕРЕКАТ >>379584 (OP)


Топ тредов
Избранное